Vous êtes sur la page 1sur 26

Distancia

Test 2VPediatría
Grupo CTO
CTO Medicina

Preguntas

1. Señale la afirmación INCORRECTA: 5. Señale la afirmación correcta respecto al test de Apgar:


1) Un niño cuadruplica su peso del nacimiento a los 2 años 1) Se debe practicar únicamente a los RN patológicos o su-
de vida y duplica su talla a los 4 años. puestamente patológicos.
2) La fontanela anterior suele cerrarse a los 12 meses y la 2) Entre los parámetros que se valoran se encuentra la frecuencia
posterior a las 6-8 semanas. respiratoria.
3) El perímetro cefálico suele igualar al perímetro torácico al año. 3) Se debe realizar al minuto y a los 5 minutos de vida.
4) La piel del RNPT está cubierta por una crema blanquecina 4) Un test de Apgar de 4 al minuto de vida implica un alto
llamada vérnix. riesgo de parálisis cerebral.
5) Las lesiones dérmicas como el milium y el eritema tóxico 5) La palidez cutánea generalizada supone una puntuación de
carecen de significado patológico. 1 en el test de Apgar.

2. Todo lo siguiente es normal en un RN, EXCEPTO: 6. Con respecto al cefalohematoma, es FALSO que:
1) Cierre del conducto de Arancio y del ductus. 1) Suele afectar a parietales con o sin fractura lineal asociada.
2) Eliminación de meconio en las primeras 48 horas de vida. 2) Es una hemorragia subperióstica cuya intensidad en ocasiones
3) Menor capacidad para concentrar la orina que a edades puede requerir transfusión y/o fototerapia.
posteriores. 3) Suele sobrepasar la línea media y líneas de sutura.
4) FC de 140 lpm y FR de 30-40 rpm. 4) La piel suprayacente suele ser normal.
5) Hemoglobina más alta y leucocitos más bajos que en un 5) La colección sanguínea aparece lentamente, por ello no se
niño de 2 años. hace manifiesto hasta varias horas postparto.

3. Recién nacido a término que presenta a las pocas horas de vida 7. Recién nacido mediante un parto de nalgas presenta la mano
cianosis e intenso distress respiratorio. A la exploración, usted derecha caída. Los dedos de dicha mano aparecen semiabier-
observa un abdomen excavado y ausculta el latido cardíaco tos y no es capaz de agarrar el dedo de su madre con esa
en el lado derecho. Respecto a la patología que sospecha, mano pero sí con la izquierda. Señale la respuesta incorrecta
señale la afirmación FALSA: respecto a la lesión obstétrica que sufre este recién nacido:
1) Se produce por alteración en el cierre de los canales pleu- 1) Se debe a una lesión en las raíces inferiores del plexo braquial.
roperitoneales posterolaterales. 2) El reflejo de Moro no estará presente en el lado derecho.
2) El diagnóstico suele hacerse por la radiografía, apreciándose 3) El parto de nalgas favorece esta situación.
asas intestinales en el tórax y desplazamiento del mediastino. 4) Puede asociarse con síndrome de Horner.
3) El cierre del defecto anatómico es considerado una urgencia 5) Este tipo de parálisis es menos frecuente que la parálisis
quirúrgica. de Erb.
4) Con frecuencia se asocia a otras malformaciones congénitas
importantes. 8. Recién nacido de 42+4 semanas de edad gestacional y 4.250 g
5) Entre las potenciales complicaciones graves destaca el desa- de peso, nacido de parto de nalgas, que presenta en la explo-
rrollo de hipertensión pulmonar y la hipoplasia pulmonar. ración caput grande, crepitación a nivel de clavícula derecha,
mano derecha caída con reflejo de prensión palmar negativo,
4. Un recién nacido, a los 5 minutos de vida presenta: F.C. 60 lati- que a las 36 horas de vida sufre un empeoramiento de su
dos por minuto, cianosis acra, esfuerzo respiratorio ausente, tono estado general, con polipnea e hipotensión arterial. ¿Cuál de
muscular inexistente y ausencia de respuesta al introducir un los siguientes datos NO esperaría encontrar en este niño?:
catéter por su nariz. La puntuación de Apgar en este niño será: 1) Masa palpable en un costado.
1) 5. 2) Anemia en el hemograma.
2) 4. 3) Cociente neutrófilos inmaduros/neutrófilos totales mayor
3) 3. de 0,16.
4) 2. 4) Ictericia.
5) 1. 5) Disminución del sodio y aumento del potasio.

CTO Medicina • C/Núñez de Balboa 115 • 28006 Madrid • Tfno.: (0034) 91 782 43 30/33/34 • E-mail: secretaria@ctomedicina.com • www.grupocto.es
1
Pediatría
9.
Test 2V Distancia
RNPT de 3 días de vida y 2300 g de peso, que presenta en la
exploración una tumoración abdominal cubierta por piel, en la
Preguntas
Grupo CTO
CTO Medicina

y tórax abombado, que mejora tras 48 horas con presión


parcial positiva al final de la espiración, pudiéndose retirar
base del cordón umbilical, fácilmente reductible, palpándose ésta. Una vez en planta, presenta irritabilidad, inquietud y
entonces un orificio herniario rodeado de bordes duros. ¿Cuál apnea, destacando en la exploración una hiperresonancia en
sería el diagnóstico más probable?: hemitórax derecho, con disminución del murmullo vesicular
1) Onfalocele. en dicho hemitórax. Lo más probable es que se trate de:
2) Granuloma umbilical. 1) Enfermedad de membrana hialina, que ha evolucionado a
3) Gastrosquisis. displasia broncopulmonar.
4) Persistencia del uraco. 2) Taquipnea transitoria del RN complicada.
5) Hernia umbilical. 3) Neumonía por Estreptococo del grupo B.
4) Síndrome de aspiración meconial asociado a neumotórax
10. RN de 36 semanas de EG, presenta, a las dos horas de vida, derecho.
polipnea de 60 rpm, retracción subesternal y subcostal con 5) Síndrome de Wilson - Mikity.
quejido espiratorio. Precisa administración de oxígeno, alcan-
zando una FIO máxima de 0.35, con lo que mejora el cuadro 14. RNPT de 29 semanas de edad gestacional, que a los 31 días
tras 6 horas de evolución. Respecto a este cuadro, señale la de vida precisa oxígeno para mantener una saturación de
afirmación que NO es cierta: oxígeno adecuada. Con respecto al cuadro que presenta este
1) Entre sus factores predisponentes destaca un parto vagi- paciente, señale la respuesta FALSA:
nal rápido o por cesárea, así como la sobrehidratación 1) Si aparecen hepatomegalia y edemas en miembros, se debe
materna. sospechar una insuficiencia cardíaca derecha.
2) Se trata de un Síndrome de Avery, o también conocido como 2) En el tratamiento es indispensable aumentar el aporte de
distress respiratorio tipo II. líquidos, que mejora la evolución del cuadro.
3) Los pacientes suelen recuperarse alrededor de las 2 semanas 3) La radiografía de este niño mostrará un patrón en esponja
de edad. característico.
4) En la Rx de tórax se observan marcas vasculares prominentes 4) Si no se encuentran antecedentes de EMH, se trata de un
con líquido en las cisuras, hiperinsuflación y, ocasionalmente, síndrome de Wilson-Mikity.
derrame pleural. 5) Habitualmente se alcanza la normalidad desde el punto de
5) Sus rasgos distintivos son su rápida recuperación y el patrón vista respiratorio en los primeros dos años.
radiológico.
15. Un recién nacido presenta taquipnea y en la radiografía de
11. El síndrome de distress respiratorio tipo I o enfermedad de tórax se observan imágenes vasculares pulmonares promi-
membrana hialina es responsable de forma directa o indirecta nentes, diafragma aplanado y líquido en las cisuras. No existe
del 50% de muertes en el período neonatal. Con respecto a hipoxemia, hipercapnia o acidosis. ¿Cuál de las siguientes
ella, señale la aseveración FALSA: enfermedades es más probable?:
1) Se manifiesta como un cuadro de distress respiratorio precoz 1) Enfermedad de la membrana hialina.
que, a menudo, es relativamente refractario a la adminis- 2) Aspiración de meconio.
tración de oxígeno. 3) Neumomediastino.
2) En el 75-80% de los casos, al estudiar el líquido amniótico 4) Taquipnea transitoria del recién nacido.
deglutido, presenta un cociente L/E menor de 2 y ausencia 5) Síndrome de Wilson-Mikity.
de fosfatidilglicerol.
3) Se caracteriza por un cuadro de hipercapnia progresiva con 16. ¿Cuál de los siguientes recursos es el más adecuado para
leve hipoxia y acidosis metabólica. tratar la hiperbilirrubinemia (11,2 mg/dl) en un lactante de 3
4) En la Rx de tórax, es característica la presencia de un patrón semanas, alimentado al pecho y con crecimiento y desarrollo
reticulogranular con broncograma aéreo y menor volumen normales?:
pulmonar. 1) La fototerapia.
5) La administración de surfactante intratraqueal en las pri- 2) La transfusión de recambio.
meras 24 h disminuye la mortalidad de la enfermedad 3) El fenobarbital.
de membrana hialina, pero no la incidencia de displasia 4) Retirar definitivamente la lactancia materna.
broncopulmonar. 5) Continuar con lactancia materna y tranquilizar a los padres.

12. RN pretérmino, que al nacer presentó un cuadro de distress 17. En un RNPT de 30 semanas, intubado por EMH y con trata-
respiratorio, que precisó ingreso en cuidados intensivos y miento antibiótico de amplio espectro por sospecha de sepsis,
aplicación de oxigenoterapia mediante presión parcial positiva a las 17 horas de vida se encuentran cifras de bilirrubina total
espiratoria (PPPE). Al 3er día de vida, se produce mejoría clínica de 20 mg/dl, con predominio de indirecta. Señale la opción
y gasométrica, permitiendo retirar la PPPE. Al 5º día de vida, verdadera:
empeora la función respiratoria y se evidencia en la gasometría 1) El tratamiento con fototerapia simple exclusivamente sería
una retención de CO2. En la exploración destaca la ausencia suficiente, pues es rápida y eficaz para este enfermo.
de cianosis, pulsos periféricos saltones, frémito palpable en 2) En este caso sería conveniente realizar una exanguinotrans-
2º espacio intercostal izquierdo. AC: soplo áspero en sístole y fusión, ante el elevado riesgo de neurotoxicidad.
diástole. ¿Cuál sería el diagnóstico más probable?: 3) El tratamiento con fenobarbital es efectivo en pocas horas,
1) Trasposición de las grandes arterias. aumentando la conjugación y excreción de bilirrubina.
2) CIV. 4) No es necesario tomar ninguna medida, pues las cifras de
3) Tetralogía de Fallot. bilirrubina están en rango normal.
4) DAP. 5) Al ser un RNPT, tiene menos riesgo de kernicterus que un
5) Coartación de aorta. RNT.

13. RN de 44 semanas de edad gestacional, con antecedentes de 18. Neonato de 2 días de vida, que presenta ictericia cutáneo-
sufrimiento fetal agudo, que en la 2ª hora de vida presenta mucosa significativa. ¿Cuál de las siguientes opciones, con
distress respiratorio con mayor tendencia a pausas de apnea respecto a las posibles causas de su ictericia, es FALSA?:

CTO Medicina • C/Núñez de Balboa 115 • 28006 Madrid • Tfno.: (0034) 91 782 43 30/33/34 • E-mail: secretaria@ctomedicina.com • www.grupocto.es
2
1)
Preguntas
Grupo CTO
CTO Medicina

Un parto traumático y un RN con múltiples hematomas a


la exploración, explican la ictericia.
Test 2V
Distancia
3)
Pediatría
El 75% de los hijos de madre diabética desarrollarán después
hipoglucemia sintomática.
2) Entre las causas más frecuentes de ictericia precoz, hay que 4) Tienen mayor incidencia de distress respiratorio.
descartar un hipotiroidismo congénito. 5) La hipoplasia de colon izquierdo es una malformación
3) El antecedente de bolsa rota durante 37 horas debe hacernos intestinal a tener en cuenta en estos niños.
sospechar una posible sepsis.
4) Una infección connatal por CMV puede debutar así. 24. Sobre la etiología de las sepsis neonatales, señale la opción
5) Si el test de Coombs directo es positivo, se trata de una correcta:
enfermedad hemolítica. 1) La Candida albicans es un patógeno frecuente en la sepsis
precoz de hijos de madres diabéticas.
19. RN de 32 semanas, con expulsión de meconio en las prime- 2) El Staphylococcus epidermidis es el agente más frecuente-
ras 24 horas, que en el 10º día de vida presenta intolerancia mente responsable de sepsis tardía.
alimentaria, vómitos biliosos, distensión abdominal y una 3) El Streptococcus agalactiae serotipo III y el E. coli serotipo
deposición sanguinolenta. En la analítica: neutropenia y K1 son los agentes que más frecuentemente producen
trombopenia con hiponatremia grave; pH 7.19; en la Rx de meningitis neonatal.
tórax se observa neumoperitoneo. ¿Cuál sería el tratamiento 4) El E. coli es el patógeno más frecuente en la sepsis precoz
más adecuado?: en nuestro medio.
1) Descompresión nasogástrica, alimentación parenteral y 5) Si se sospecha Listeria monocytogenes, se debe emplear
antibioterapia sistémica. Cefotaxima en vez de Gentamicina.
2) Piloromiotomía extramucosa.
3) Enema de suero salino isotónico. 25. RN de 35 semanas, con antecedente de bolsa rota de más de
4) Enema de contraste yodado. 24 horas, presenta, a las 8 horas de vida, hipotermia, mala
5) Resección del intestino necrosado y colostomía de descarga. perfusión periférica, letargia y taquipnea. En la analítica des-
taca leucocitos 32.430, con índice infeccioso de 0.32; señale
20. Un RN pretérmino, con antecedentes de sufrimiento fetal, lo FALSO sobre la sepsis del recién nacido:
comienza con distensión abdominal importante, vómitos y 1) Las manifestaciones neurológicas predominan en la sepsis
deposiciones hemorrágicas. En la Rx de abdomen se observa de inicio precoz.
edema de asas intestinales, con un patrón en “miga de pan” y 2) Los agentes causales más frecuentes son el estreptococo del
presencia de gas en la pared intestinal. Respecto a esta pato- grupo B y E. coli.
logía, señale cuál de las siguientes afirmaciones es FALSA: 3) Se debe sospechar en todo neonato con temperatura ines-
1) Las situaciones de hipoxia y bajo gasto predisponen a esta table y rechazo del alimento.
patología. 4) La mortalidad es mayor en aquella que aparece antes de la
2) La primera manifestación clínica es la aparición de distensión semana de vida.
abdominal. 5) El tratamiento empírico adecuado es ampicilina y gentami-
3) La presencia de neumatosis intestinal es indicación de cirugía cina, durante al menos 7 días.
urgente.
4) La supresión de la alimentación oral y el reposo intestinal 26. ¿Cuál sería la pauta más correcta a llevar a cabo en el RN de
con descompresión mediante SNG es necesario. madre con la siguiente serología: Ag s + frente al virus de la
5) Se han involucrado en su etiología a diversos patógenos hepatitis B?:
como E.coli, C.perfringens, Rotavirus y S. epidermidis 1) Vacuna.
2) Gammaglobulina.
21. Todos los siguientes grupos de recién nacidos tienen riesgo 3) Interferón Alfa.
de hipoglucemia neonatal salvo uno, señálelo: 4) Vacuna + Gammablobulina.
1) Hijos de madre diabética. 5) Vacuna+ gammaglobulina+ interferón.
2) Niños con convulsiones neonatales.
3) Neonatos con crecimiento intrauterino retardado y prematuros. 27. Señale la afirmación INCORRECTA, respecto a las alteracio-
4) Enfermedades metabólicas como la galactosemia o las glu- nes endocrinológicas que pueden detectarse en el período
cogenosis. neonatal:
5) Neonatos gravemente enfermos, independientemente de 1) El panhipopituitarismo puede manifestarse como micropene
la patología de base. e hipoglucemia.
2) El hipoparatiroidismo puede manifestarse clínicamente como
22. RN, hijo de madre ADVP, comienza, a las 36 horas de vida, tetania.
con irritabilidad, temblores y mala tolerancia. Todas las si- 3) La hemorragia adrenal aguda puede verse tras partos de
guientes afirmaciones son ciertas, EXCEPTO: nalgas, partos traumáticos o infecciones fulminantes.
1) Tienen mayor riesgo de desarrollar anomalías congénitas. 4) El RN con hipopituitarismo o déficit aislado de GH suele
2) La clínica del síndrome de abstinencia comienza antes en tener una talla y un peso al nacimiento menor de -2DS de
hijos de madre adictas a heroína que a metadona. la media.
3) Los hijos de adictas a metadona tienen más riesgo de sufrir 5) La insuficiencia suprarrenal se puede manifestar por vómitos,
convulsiones. diarrea, deshidratación, hiperpotasemia e hiponatremia.
4) Está indicado tratamiento con morfina oral.
5) La incidencia de EMH e hiperbilirrubinemia en RNBP de 28. RNT, sin antecedentes de interés, al que se le realizó profilaxis
madres adictas a opiáceos es menor que en la población ocular y antihemorrágica a los pocos minutos del nacimien-
general. to, presenta, a las 48 horas de vida, sangrado por el cordón
umbilical y sangre fresca en heces. Señale la INCORRECTA:
23. Con respecto al hijo de madre diabética, NO es cierto que: 1) No estará indicado administrar vitamina K, porque ya ha
1) Puede expresarse clínicamente retraso de la evacuación de recibido una dosis de profilaxis.
meconio. 2) Un factor de riesgo para la aparición de este cuadro es el
2) Si la madre sufre trastornos vasculares, el niño puede ser tratamiento materno con fenitoína.
CIR. 3) Estaría indicado realizar un estudio de coagulación.

CTO Medicina • C/Núñez de Balboa 115 • 28006 Madrid • Tfno.: (0034) 91 782 43 30/33/34 • E-mail: secretaria@ctomedicina.com • www.grupocto.es
3
Pediatría
4)
5)
Test 2V Distancia
El cuadro puede deberse a una sepsis con CID.
La enfermedad hemorrágica por déficit de vitamina K de
4)
Preguntas
Grupo CTO
CTO Medicina

La leche materna es la más adecuada, añadiéndose suple-


mentos de proteínas, calcio y fósforo.
comienzo tardío (>1 semana) puede estar asociada a una 5) En los prematuros que necesitan fluidoterapia intravenosa,
hepatitis neonatal o atresia biliar. hay que tener en cuenta que las pérdidas insensibles son
menores que en los nacidos a término, por lo que los aportes
29. Con respecto a la anemia en el período neonatal, NO es cierto líquidos deben ser menores.
que:
1) La cifra de hemoglobina en el RNT al nacimiento es aproxima- 34. Recién nacido a término, que traen a Urgencias sus padres
damente de 14-20 g/dl y en el RNPT, 1-2 gramos menos. a los 11 días de vida, por “enrojecimiento y supuración” del
2) Una de las causas más frecuentes de anemia al nacer es la ombligo, que han detectado ese mismo día. El niño no ha
enfermedad hemolítica. tenido fiebre, tiene buen estado general y come bien, según
3) Una transfusión fetomaterna puede diagnosticarse por la refieren. En la exploración física destaca una tumoración
prueba de Kleihauer-Betke en la sangre materna. ligeramente prominente en el ombligo, rosada, blanda y
4) Una pérdida aguda de sangre inicialmente puede cursar con granular a la palpación, con secreción serosa y no purulenta,
hemoglobina normal y sin hepatoesplenomegalia, pero con con normalidad de la piel de alrededor. ¿Cuál sería su actitud
signos incipientes de shock. diagnóstico-terapeútica?:
5) Existe un descenso fisiológico en las cifras de Hb, que se 1) Realizar hemograma, PCR y recoger muestra para hemo-
produce en la 6ª semana de vida en los RNT y hacia la 8-12ª cultivo, iniciando antibioterapia hasta el resultado de éste.
semana en los RNPT. 2) Realizar biopsia y decidir el tratamiento una vez tenga el
informe anatomopatológico.
30. RN de madre 0- y padre A+, con embarazo no controlado. 3) Cauterizar con nitrato de plata en varias sesiones hasta llegar
Con respecto a los cuadros clínicos que puede presentar el a la base.
neonato, señale la opción FALSA: 4) Esperar hasta el año de edad, porque lo más probable es
1) Si el Coombs indirecto para la incompatibilidad Rh es que la formación vaya desapareciendo espontáneamente
negativo, se debe realizar profilaxis con gammaglobulina hasta entonces.
hacia la semana 28 de gestación, y en las primeras 72 horas 5) Extirpación quirúrgica completa, por el riesgo de degene-
postparto, si el RN es Rh+. ración maligna.
2) Puede ser necesaria la transfusión intrauterina de hematíes,
si la hemólisis es intensa. 35. Señale la afirmación FALSA, de entre las siguientes, respecto
3) En la incompatibilidad de grupo puede haber afectación a la ictericia fisiológica:
del primer hijo. 1) Las cifras máximas de bilirrubina suelen alcanzarse a los
4) Si el Coombs directo es positivo frente al Ag Rh, es impres- 3-4 días.
cindible administrar gammaglobulina a la madre. 2) La ictericia en los RNPT suele ser de inicio algo más tardío
5) La aparición de una DO 450 en zona III en el análisis es- y más prolongada que en los RNT.
pectrofotométrico del líquido amniótico indica un riesgo 3) Puede comenzar en las primeras 24 horas de vida en el RNT.
fetal alto. 4) Su duración suele ser inferior a 10-15 días.
5) En los RNPT pueden alcanzarse cifras de hasta 14 mg/dl.
31. Señale la opción FALSA, con respecto a la policitemia neonatal:
1) Puede ser causa de ictericia en la primera semana de 36. RN a término al que se le determina el valor de TSH en san-
vida. gre, obtenida mediante la prueba del talón, el día 3 de vida.
2) Es más frecuente en los hijos de madre diabética. El valor de la TSH aparece elevado. ¿Qué actitud considera
3) Se diagnostica con un hematocrito capilar > 65%. más adecuada?:
4) Puede complicarse con insuficiencia cardíaca. 1) Repetir la prueba dentro de un mes.
5) Es más frecuente en los RN con bajo peso para su edad 2) Sospechar hipertiroidismo e iniciar tratamiento con antiti-
gestacional por insuficiencia placentaria. roideos.
3) Tranquilizar a la madre porque se trata de algo habitual en
32. Un RNPT de bajo peso al nacer, presenta un cuadro com- los RN.
patible con una crisis motora focal. En la TC craneal apa- 4) Sospechar hipotiroidismo congénito e iniciar tratamiento
recen ventrículos ligeramente dilatados y calcificaciones sustitutivo con hormona tiroidea.
periventriculares. ¿Qué patología es más probable que 5) Ingresar al RN en observación.
presente este RN?
1) Sífilis congénita. 37. Niña de 20 días de vida, que acude por presentar dificultad
2) Toxoplasmosis congénita. para la alimentación, con inactividad, apatía y escaso llanto,
3) Rubéola congénita. así como respiración ruidosa. Entre sus antecedentes destaca
4) Varicela congénita. haber precisado ingreso hospitalario por ictericia. En la ex-
5) Infección prenatal por citomegalovirus. ploración presenta apatía, abdomen voluminoso, con hernia
umbilical, edema de miembros con piel fría y moteada. La
33. La premadurez es cada vez más frecuente y se asocia a un fontanela anterior mide 3x3 cm y la posterior 2x2. Presenta
aumento de la morbimortalidad. ¿Cuál de las siguientes res- baja implantación del pelo y macroglosia. El recuento y fór-
puestas es correcta acerca de los cuidados específicos que mula sanguínea son normales, sin alteraciones en los datos
deben recibir estos recién nacidos?: bioquímicos. El diagnóstico más probable es:
1) Es recomendable introducirles en incubadora, a una tem- 1) Hipertiroidismo congénito.
peratura de 36.5-37ºC, coincidiendo con la temperatura 2) Fenilcetonuria.
corporal materna. 3) Hipotiroidismo congénito.
2) La alimentación debe iniciarse por sonda nasogástrica en 4) Raquitismo.
todos los menores de 32 semanas de edad gestacional. 5) Hiperplasia suprarrenal congénita por déficit de 21-hidroxilasa.
3) La oxigenoterapia es una práctica rutinaria en los prematuros,
por el riesgo de desarrollar un cuadro de distrés respiratorio 38. Sobre las diferencias entre leche de vaca y leche materna,
en las primeras 72 horas de vida. señale la opción correcta:

CTO Medicina • C/Núñez de Balboa 115 • 28006 Madrid • Tfno.: (0034) 91 782 43 30/33/34 • E-mail: secretaria@ctomedicina.com • www.grupocto.es
4
1)
2)
Preguntas
Grupo CTO
CTO Medicina

La leche de madre es más rica en grasas que la de vaca.


El flúor es más abundante en la leche de vaca.
Test 2V
Distancia Pediatría
una infección respiratoria alta (como el resto de la familia).
¿Cuál es el diagnóstico más probable?:
3) Las seroproteínas se encuentran en mayor porcentaje en la 1) Epiglotitis por H. influenzae.
leche de vaca. 2) Crup viral.
4) La enfermedad hemorrágica del RN es más frecuente en los 3) Crup espasmódico.
niños que se alimentan con fórmula artificial. 4) Difteria.
5) Los ácidos grasos poliinsaturados existen en mayor concen- 5) Cuerpo extraño en laringe.
tración en la leche de madre.
44. Niño de 3 años de edad, atendido 3 días antes en Urgencias
39. Acude a su consulta una madre con su hija de 10 años preocu- por un cuadro típico de laringotraqueítis vírica. La madre
pada porque dice que su hija “siempre ha sido la más bajita consulta de nuevo por empeoramiento de la tos y la fiebre,
de toda la clase”. Su talla actual se encuentra en el percentil decaimiento, y dificultad respiratoria en las últimas 24 horas.
2 de la población general. Su talla genética está situada en el En la exploración física el niño tiene regular estado general,
percentil 3. Su desarrollo sexual es el que corresponde para con dificultad respiratoria llamativa y estridor inspiratorio
su edad y su edad ósea es de 9 años. ¿Qué situación cree y espiratorio. En la auscultación se oyen roncus dispersos.
usted que presenta la niña?: Señale la respuesta correcta acerca del caso:
1) Un hipocrecimiento por deficiencia de la hormona de 1) Hay que sospechar una neumonía, siendo el antibiótico
crecimiento. empírico indicado en esta edad la amoxicilina.
2) Un hipocrecimiento por un síndrome de Turner. 2) Hay que administrar una dosis de salbutamol inhalado, y si
3) Un hipocrecimiento por un hipotiroidismo congénito. no mejora, realizar una radiografía de tórax.
4) Un retraso constitucional del crecimiento y del desarrollo. 3) Lo más importante es asegurar la permeabilidad de la vía
5) Una talla baja familiar. aérea, con intubación endotraqueal hasta que sea efectivo
el tratamiento etiológico.
40. Niño de 4 meses y 5 Kg de peso, que presenta deposiciones 4) Algunos casos producen síndrome de shock tóxico.
líquidas frecuentes (15/día), desde hace 72 horas, con febrí- 5) Las complicaciones son relativamente frecuentes, siendo la
cula y mala tolerancia oral. Peso 4.5 Kg, TA: 75/60. Sequedad más importante el desarrollo de bronquiectasias.
de piel y mucosas, irritabilidad, pulsos palpables. Resto de
exploración normal. La analítica muestra: Na+: 170 mEq/l; 45. ¿Cuál de los siguientes apoyaría el diagnóstico de crisis de
K+: 4.0 mEq/l; Cl-: 100 mEq/l; pH 7.10; bicarbonato 9 mEq/l broncoespasmo frente al de bronquiolitis en un lactante de
y PCO2: 18. Señale la opción correcta: 11 meses que presenta febrícula, tos y dificultad respiratoria
1) Reposición rápida con suero salino fisiológico a 20 cc/Kg, desde hace 24 horas?:
ante la situación de shock de este paciente. 1) Antecedentes personales de episodios similares.
2) Corrección lenta en 72 horas, debido al riesgo neurológico. 2) Comienzo progresivo, con signos iniciales de infección de
3) Corrección con suero glucosado en 24 horas. vías respiratorias altas.
4) Serán llamativos los signos de depleción vascular. 3) Ausencia de mejoría tras la administración de broncodila-
5) Es la forma de deshidratación más frecuente. tadores nebulizados.
4) Madre fumadora.
41. Señale cuál de las siguientes afirmaciones sobre el raquitismo 5) Signos de hiperinsuflación pulmonar en la radiografía de
NO es cierta: tórax.
1) La existencia de craneotabes parietal en un RN debe ha-
cernos sospechar la existencia de un raquitismo por déficit 46. Con respecto a la bronquiolitis aguda, es FALSO que:
de vitamina D. 1) El adenovirus puede asociarse a complicaciones a largo plazo,
2) Las alteraciones radiológicas preceden en la mayoría de los como bronquiolitis obliterante y síndrome de hiperclaridad
casos a las manifestaciones clínicas. pulmonar unilateral.
3) Las necesidades diarias de vitamina D en un RNT son de 2) La fuente de la infección viral suele ser un miembro de la
400 UI. familia con una enfermedad respiratoria leve.
4) En la Rx se observan metáfisis ensanchadas con imagen “en 3) La exploración muestra un lactante taquipneico con dificultad
copa”. respiratoria y sibilancias espiratorias.
5) El raquitismo por déficit de vitamina D es más frecuente en 4) Casi la mitad de los lactantes que padecieron bronquiolitis
niños de raza negra. presentará en un futuro hiperreactividad de las vías aéreas
durante la infancia.
42. Las variantes de la normalidad son las causas más frecuentes 5) Son raras las complicaciones bacterianas, tales como bron-
de talla por debajo de 2 desviaciones estándar. Con respecto coneumonía u OMA.
a ellas, NO es cierto que:
1) Los niveles de hormona del crecimiento son normales. 47. Niño de 6 años, diagnosticado de faringitis aguda hace 6 días,
2) En la talla baja de origen genético, la edad ósea es menor comienza de forma brusca con fiebre alta, dificultad para
que la cronológica. tragar, intenso distress respiratorio e importante dolor de
3) En el retraso constitucional del crecimiento, la edad ósea garganta. En la exploración se observa una masa en la pared
coincide con la estatural y ambas son menores que la cro- posterior de la faringe. El diagnóstico más probable sería:
nológica. 1) Faringolaringitis aguda.
4) El pronóstico de talla es normal o ligeramente disminuido 2) Cuerpo extraño.
en el retraso constitucional. 3) Epiglotitis aguda.
5) En la talla baja familiar, la pubertad aparece dentro de los 4) Traqueítis bacteriana.
límites de la normalidad. 5) Absceso retrofaríngeo.

43. Niño de 9 meses, bien vacunado, que a la 1 de la madruga- 48. Niño de 6 años que, estando previamente bien, inicia de
da, en pleno invierno, acude a Urgencias por despertarse de forma progresiva una tos seca con dificultad respiratoria. En
pronto con tos perruna, fiebre de 38oC. y agitación. No babea la exploración está afebril, con ligera cianosis periorificial,
y toma algunos líquidos. En los dos días pasados ha tenido tiraje subcostal y supraesternal con sibilancias audibles a

CTO Medicina • C/Núñez de Balboa 115 • 28006 Madrid • Tfno.: (0034) 91 782 43 30/33/34 • E-mail: secretaria@ctomedicina.com • www.grupocto.es
5
Pediatría
Test 2V Distancia
distancia. En la auscultación pulmonar presenta espiración
alargada con hipoventilación bilateral y sibilancias espiratorias
4)
Preguntas
Grupo CTO
CTO Medicina

La mortalidad aumenta si los síntomas están presentes du-


rante más de 48 horas.
difusas. La Rx de tórax muestra hiperinsuflación pulmonar 5) La localización más frecuente es la exclusivamente ileal.
con costillas horizontalizadas y diafragma aplanado, así como
un ligero aumento de la trama broncovascular. El diagnóstico 53. Con respecto a la invaginación intestinal, señale la opción
más probable será: FALSA:
1) Bronquiolitis. 1) Si este cuadro aparece en niños mayores de 6 años, debemos
2) Tos psicógena. descartar un linfoma intestinal.
3) Crisis asmática. 2) Si no se diagnostica precozmente, aparecen las deposiciones
4) Neumonía por Mycoplasma spp. en jalea de grosella.
5) Crup espasmódico. 3) La técnica diagnóstica de elección es la ecografía abdominal.
4) Hasta un 45% de los casos tratados con enema recidivan.
49. Kevin es un niño de cuatro años, que acude a urgencias del 5) El signo del muelle enrollado nos ayuda a establecer el
hospital en el que Vd. está de guardia a las dos de la madru- diagnóstico.
gada. La madre refiere que desde la tarde se queja de dolor
de garganta y presenta fiebre de 39,5ºC. Cuando Vd. lo ve, 54. Recién nacido que presenta inmediatamente tras el nacimiento
está sentado, con la boca abierta, presentando salivación exceso de secreciones orales con el abdomen llamativamente
intensa. Con respecto al proceso que Vd. sospecha, señale distendido y timpanizado. Ha nacido a la semana 35 de ges-
la opción FALSA, de entre las siguientes: tación tras cesárea por polihidramnios materno. ¿Cuál de las
1) Es probable que Kevin no haya completado el calendario respuestas es INCORRECTA acerca de la patología que debe
vacunal. sospechar?:
2) Este cuadro clínico no suele precisar intubación endotraqueal. 1) Para confirmar el diagnóstico lo más útil sería hacer una
3) En la Rx lateral de faringe se podrá apreciar la epiglotis radiografía de tórax.
aumentada de tamaño. 2) Son excepcionales las secuelas después del tratamiento
4) El tratamiento farmacológico de elección es la cefotaxima iv. quirúrgico.
5) Pueden aparecer como complicación otras infecciones, 3) El tratamiento constituye una urgencia quirúrgica, siendo
como la otitis media o la neumonía. conveniente que el niño permanezca en decúbito prono
con aspiración gástrica hasta la intervención.
50. El pilar del tratamiento de la fibrosis quística es la terapia 4) Es conveniente descartar cardiopatías estructurales asociadas
pulmonar, cuyo objetivo principal es eliminar las secreciones y antes de la cirugía.
controlar las infecciones. Señale la que le parece INCORRECTA 5) El 50% de los pacientes tienen otras anomalías congénitas
de las siguientes afirmaciones: asociadas.
1) La fisioterapia respiratoria está indicada de 1 a 4 veces
al día. 55. Niña de 15 meses consulta por estancamiento de la curva
2) La antibioterapia vía oral está indicada siempre que haya ponderal. Desde hace unos meses presenta heces más blan-
signos o síntomas de exacerbación respiratoria, eligiéndose los das, pastosas, fétidas y pegajosas. Presenta mal carácter,
fármacos de forma empírica, pues la realización de cultivos anorexia y decaimiento. No vómitos ni dolor abdominal.
en estos pacientes es poco rentable. Se le realiza un hemograma con Hb: 10 g/dl, Fe sérico 8
3) La tobramicina en aerosol consigue disminuir el número mg/dl, ferritina 4 ng/ml. Test de Van de Kamer con grasas
de hospitalizaciones de los pacientes colonizados por P. totales de 7 g/día. ¿Cuál sería la actitud más adecuada en
aeruginosa y mejorar su función pulmonar después de 6 este paciente?:
meses de tratamiento. 1) Retirar el gluten de la dieta y después realizar biopsia intes-
4) Las dosis de antibióticos orales en estos pacientes suelen ser tinal.
2-3 veces mayores que las convencionales. 2) Mantener el gluten al menos dos años, y después hacer
5) Los corticoides sistémicos se utilizan para tratar la aspergilosis biopsia intestinal.
alérgica. 3) Realizar biopsia intestinal, y si es patológica, retirar el gluten
de la dieta.
51. El primogénito de una pareja sufre en el período neonatal íleo 4) Retirar definitivamente el gluten de la dieta, sin necesidad
meconial. ¿Cuál es el método de diagnóstico principal que de realizar biopsia.
llevaría a cabo para establecer el diagnóstico en la enfermedad 5) Instaurar dieta astringente y calmar a los padres.
subyacente más probable?:
1) Test del tripsinógeno inmunorreactivo. 56. En la biopsia del intestino delgado de un paciente celiaco,
2) Test de meconio. encontraría:
3) Estudio molecular genético directo. 1) Inclusiones PAS+.
4) Test de cloro en sudor. 2) Escaso infiltrado linfocitario.
5) Estudio genético de vínculo o cercanía. 3) Patrón digitiforme de la mucosa.
4) Atrofia subtotal de las vellosidades con hiperplasia de las
52. Varón de 7 meses, que comienza de forma brusca con crisis de criptas.
llanto intenso, acompañadas de encogimiento de miembros 5) Biopsia patognomónica.
inferiores y palidez cutánea llamativa. Entre una crisis y otra, el
niño permanece somnoliento y decaído. En la exploración física 57. Señale la afirmación FALSA, al hablar de la enfermedad
aparece distensión abdominal y se palpa una masa en hipocondrio celíaca:
derecho. Señale la afirmación FALSA sobre este cuadro: 1) La lesión intestinal es mayor en la porción proximal del
1) Es la causa más frecuente de obstrucción intestinal entre los intestino delgado.
3 meses y 6 años. 2) El 10% de los niños estudiados por retraso de crecimiento
2) Sólo en un 2-5% se ha podido encontrar una causa orgánica son portadores.
que lo favorezca. 3) La positividad de los Acs se relaciona con la actividad de la
3) En la Rx de abdomen se puede encontrar un silencio aéreo enfermedad.
en hipocondrio y flanco derecho. 4) Los Acs antigliadina son los más sensibles y específicos.

CTO Medicina • C/Núñez de Balboa 115 • 28006 Madrid • Tfno.: (0034) 91 782 43 30/33/34 • E-mail: secretaria@ctomedicina.com • www.grupocto.es
6
5)
Preguntas
Grupo CTO
CTO Medicina

Los pacientes no tratados tienen más predisposición a pa-


decer linfomas en la edad adulta.
Test 2V
Distancia
1)
Pediatría
Sustituir su fórmula artificial normal por un preparado a base
de leche sin lactosa.
2) Continuar con su fórmula artificial y ver evolución.
58. Paciente varón, con síndrome de Down de dos meses y medio 3) Alternar unas tomas con su fórmula artificial y otras con un
de edad, que refiere tos nocturna. La madre cuenta que, en hidrolizado de proteínas.
ocasiones, se pone muy rígido, con la cabeza hiperextendida. 4) Reemplazar su fórmula artificial por una fórmula de soja.
Estos episodios le ocurren sobre todo un rato después de las 5) Reemplazar su fórmula artificial por un hidrolizado pro-
tomas. Con respecto al cuadro que Vd. sospecha, señale la teico.
opción FALSA:
1) En el hemograma podemos encontrar una anemia ferropénica. 62. Niño de 13 meses, que tras padecer una gastroenteritis aguda
2) Esta entidad es más frecuente en varones, y aparece entre hace una semana, presenta una diarrea líquida que produce
1/300 y 1/1000 niños, de forma acentuada. eritema en la zona del pañal. Al explorar al niño advierte un
3) La prueba diagnóstica de elección es el esofagograma con aumento de los ruidos hidroaéreos. Respecto a esta patología
contraste de bario. qué opción es incorrecta:
4) En los casos leves, puede no ser necesario el tratamiento 1) El pH fecal suele ser menor de 7.
farmacológico. 2) La prueba del Clinitest es positiva en la malabsorción de
5) Puede debutar como neumonías de repetición por aspira- hidratos de carbono del tipo lactosa.
ciones. 3) La causa más frecuente de deficiencia secundaria de lactasa
es la diarrea infecciosa.
59. Varón de 7 meses de edad, que acude a la consulta por estre- 4) En la intolerancia secundaria a hidratos de carbono, la
ñimiento crónico, intermitente, desde el nacimiento, que ha exclusión de disacáridos es permanente.
sido resuelto con dificultad por la familia con distintas ayudas. 5) Una nutrición precoz con dieta normal en pacientes que
La madre refiere que permanece sin hacer deposición hasta han sufrido una GEA evita su aparición.
1 semana y que ésta es de características caprinas. Entre los
antecedentes personales destaca expulsión de meconio a las 63. Niño de 3 semanas presenta vómitos alimentarios, no
72 horas de vida. De la exploración física cabe señalar: peso biliosos y abundantes con la mayoría de las tomas, desde
y talla en percentil 3, distensión abdominal moderada y pal- hace tres días. En la exploración se observa dudosa masa
pación de masa fecal en fosa ilíaca derecha. En el tacto rectal de 2 cm en hipocondrio derecho. ¿Cuál es su diagnóstico
no se encuentran heces en la ampolla rectal. Con respecto al de sospecha?:
diagnóstico más probable de este paciente, es cierto que: 1) Hernia hiatal.
1) La existencia de diarrea en un caso similar permitiría excluir 2) Mala técnica alimentaria.
un megacolon congénito. 3) Insuficiencia suprarrenal.
2) La alteración más característica en el enema opaco es un 4) Estenosis hipertrófica de píloro.
megarrecto asociado a una dilatación del colon. 5) Atresia duodenal.
3) En la manometría anorrectal se detecta una relajación del
esfínter anal interno. 64. El dolor abdominal recurrente es un motivo de consulta fre-
4) El diagnóstico definitivo se realizaría mediante una biopsia cuente en los niños de la etapa preescolar y escolar. Señale
rectal. la opción correcta acerca de esta entidad:
5) El tratamiento de elección es dietético y mediante el uso de 1) En la mayoría de los menores de 2 años no se llega a iden-
medicamentos laxantes. tificar una causa orgánica.
2) Los casos en que el dolor es “funcional” son de mejor
60. Varón de 14 meses que, desde hace aproximadamente 2 me- pronóstico, mejorando por lo general en cuanto los es-
ses, realiza 4 deposiciones al día, líquidas, no malolientes ni tudios se completan sin encontrarse ninguna patología.
brillantes, y sin olor ácido, que no contienen sangre ni pus, 3) La asociación del dolor a las comidas, y su presencia por
pero en las que, a simple vista, se observan restos vegetales la noche, son dos patrones que indican casi con seguridad
sin digerir. El niño no ha perdido peso, ni se encuentra mal la existencia de una enfermedad orgánica.
por culpa de la diarrea. La exploración física es normal, con 4) La prevalencia de la infección crónica por H. pylori en estos
peso y talla en el percentil 50, sin signos de deshidratación, niños es más frecuente que en la población general.
distensión abdominal ni eritema perianal. El estudio de heces 5) Aunque la historia clínica y la exploración física indiquen
y sangre es normal. ¿Cuál es la actitud que tomaría con este un origen funcional, es necesario siempre realizar pruebas
paciente?: complementarias para descartar organicidad.
1) Realizar biopsia intestinal, y si es patológica, retirar el gluten
de la dieta. 65. Niño de 2 años que, estando previamente bien, comienza con
2) Tranquilizar a los padres, y aconsejar una dieta sin limita- sangrado rectal, sin acompañarse de dolor. Para descartar un
ciones, adecuada para la edad del niño. divertículo de Meckel, se le realiza una gammagrafía. Señala
3) Realizar una curva de sobrecarga con lactosa, y si es pato- la respuesta verdadera:
lógica, retirar el consumo de la misma. 1) La prueba más indicada en este paciente habría sido una
4) Investigar antecedentes familiares y personales de patología Rx de abdomen.
respiratoria y realizar test del sudor. 2) El estudio de contraste con Bario es de gran utilidad.
5) Instaurar dieta astringente, con restricción de leche y pro- 3) La gammagrafía con Tc99 es la técnica más sensible indepen-
ductos lácteos, y pobre en grasa. dientemente de que exista mucosa ectópica.
4) El tratamiento en este paciente sería únicamente sintomático.
61. Niño de 2 meses de edad, que ha comenzado con lactancia 5) La captación del isótopo aumenta si administramos previa-
artificial unas horas antes, y que actualmente presenta vómi- mente cimetidina, glucagón o gastrina.
tos, diarrea líquida y leve hinchazón de labios y párpados, así
como un exantema urticariforme en tronco y extremidades. 66. ¿Cuál de las siguientes características es más frecuente y peculiar
Como único antecedente de interés en la familia, destaca el de los vómitos por reflujo gastroesofágico (calasia) en el niño?
padre con asma extrínseca y atopia. La actitud terapéutica 1) Asociación con ferropenia.
más adecuada en este paciente sería: 2) Relación con la ingesta.

CTO Medicina • C/Núñez de Balboa 115 • 28006 Madrid • Tfno.: (0034) 91 782 43 30/33/34 • E-mail: secretaria@ctomedicina.com • www.grupocto.es
7
Pediatría
3)
4)
Test 2V
Repercusión sobre el crecimiento.
Distancia

Comienzo neonatal y mejoría con la edad.


Preguntas

1)
Grupo CTO
CTO Medicina

El reflujo vesicoureteral primario se debe a una anomalía


congénita de la unión ureterovesical.
5) Asociación con síntomas de broncoconstricción. 2) En los casos de duplicación ureteral, el reflujo es más fre-
cuente en el uréter que se introduce en la vejiga en el punto
67. Varón de 5 años que, desde hace una semana, presenta más alto y lateralizado.
un cuadro de varicela. En la actualidad, la mayoría de las 3) En el reflujo de grado I y II el 80% de los casos se resuelven
lesiones están en fase de costra y se encuentra afebril. De espontáneamente.
forma brusca, presenta vómitos persistentes, proyectivos, 4) En el reflujo de grado IV y V la cirugía debe demorarse, bajo
con deterioro del nivel de conciencia y alteraciones del profilaxis antibiótica, hasta asegurarse plenamente que el
comportamiento, volviéndose agresivo. En la exploración reflujo no desaparece.
sólo llama la atención una discreta hepatomegalia. En la 5) Los resultados de la cirugía son excelentes.
analítica presenta hipoglucemia, elevación importante de la
GOT, GPT, LDH y CPK, aumento del amonio sérico. Señale 73. La causa más frecuente de HTA en la infancia es:
cuál será el diagnóstico más probable: 1) Coartación de aorta.
1) Encefalitis postvaricelosa. 2) Hiperplasia suprarrenal congénita.
2) Hepatitis A aguda. 3) HTA esencial.
3) Síndrome de Reye. 4) Nefropatía por reflujo.
4) Encefalitis herpética. 5) Trombosis de la arteria renal.
5) Debut de una metabolopatía, pendiente de filiar.
74. Un niño de 12 años y 5 meses se presenta en el servicio
68. Sobre el síndrome de Reye, es FALSO: de Urgencias con dolor inguinal y escrotal intenso, que
1) Debe diferenciarse de algunas metabolopatías, como los comenzó de forma brusca después de un partido de fútbol.
defectos de oxidación de ácidos grasos. El niño está afectado y se observa tumefacción y dolor en
2) Su cuadro clínico se caracteriza por el desarrollo de focalidad hemiescroto derecho, no se detecta reflejo cremastérico
neurológica. y el teste derecho está elevado y rotado. Señale la opción
3) Existe una disfunción mitocondrial subyacente. FALSA, respecto a este cuadro:
4) Parece asociarse al uso de aspirina en las infecciones virales. 1) Habitualmente ocurre en niños mayores de doce años.
5) En el tratamiento, las medidas antiedema cerebral son 2) La torsión del apéndice testicular es más frecuente entre los
importantes. 2 y los 11 años.
3) En niños mayores de 13 años, debe hacerse el diagnóstico
69. Cuando se diagnostica de fimosis a un lactante, al comentarles diferencial con la epididimitis.
sobre el proceso a sus padres tenemos que tener en cuenta que: 4) El tratamiento óptimo es la cirugía inmediata.
1) Hasta los dos años se puede tratar de un proceso fisiológico. 5) La torsión testicular puede producirse en el período neonatal,
2) Es un hallazgo poco frecuente. pero en estos casos no suele haber ningún riesgo de perder
3) Se realiza circuncisión solo en caso de existir motivos religiosos. las gónadas.
4) Las adherencias peneanas y la fimosis se trata de una misma
patología. 75. Lactante varón de 10 meses de edad al que en un examen
5) En la actualidad el uso de pomadas de corticoides evita la rutinario de salud se le descubre una masa en flanco izquierdo,
circuncisión por motivos médicos. dura, que sobrepasa línea media. En la ecografía abdominal
dicha masa se corresponde con un tumor sólido localizado
70. Señale la respuesta FALSA sobre la estenosis pielocalicial en la glándula suprarrenal izquierda. ¿Cuál de las siguientes
congénita: afirmaciones es cierta?:
1) Es la causa más frecuente de obstrucción de la vía urinaria 1. La edad inferiro al año empeora el pronóstico.
en la infancia. 2. Si presentara metástasis hepáticas estaría contraindicado el
2) Es dos veces más frecuentes en varones que en mujeres. tratamiento quirúrgico.
3) En el 10% de los casos la estenosis es bilateral. 3. Debe realizársele un aspirado de médula ósea como parte
4) Los casos detectados prenatalmente como hidronefrosis del estudio de extensión.
fetal, deben ser estudiados postnatalmente con ecografía y 4. El tumor que presenta es más frecuente en pacientes con
además con cistouretrografía miccional retrógrada. hemihipertrofia.
5) El tratamiento de elección consiste en realizar una pielo- 5. El empleo de la gammagrafía con metayodobencilguanidina
plastia, aunque los resultados son mediocres y sólo el 50% ha sido abandonado por técnicas más específicas.
son satisfactorias.
76. Con respecto a los factores pronósticos del neuroblastoma,
71. Señale cuál de las siguientes afirmaciones sobre el raquitismo señale la opción FALSA:
NO es cierta: 1) El diagnóstico en menores de un año mejora el pronóstico.
1) La existencia de craneotabes parietal en un RN debe ha- 2) La localización abdominal es la de peor pronóstico.
cernos sospechar la existencia de un raquitismo por déficit 3) La amplificación del oncogén myc es un factor de mal pro-
de vitamina D. nóstico.
2) Las alteraciones radiológicas preceden en la mayoría de los 4) Las metástasis óseas son de peor pronóstico que las de
casos a las manifestaciones clínicas. hígado, piel o médula ósea.
3) Las necesidades diarias de vitamina D en un RNT son de 5) La aparición de un síndrome de opsoclono mioclono em-
400 UI. peora el pronóstico.
4) En la Rx se observan metáfisis ensanchadas con imagen “en
copa”. 77. Niño de tres años de edad, que es remitido al hospital porque,
5) El raquitismo por déficit de vitamina D es más frecuente en en una revisión rutinaria, su pediatra le ha palpado una masa
niños de raza negra. abdominal indolora en flanco derecho. La madre refiere que al
nacer “era muy gordo y tenía las tripas fuera”. Con respecto
72. En relación con el reflujo vesicoureteral en la infancia, señale al tumor que Vd. sospecha, son ciertas todas las siguientes,
la respuesta FALSA: EXCEPTO:

CTO Medicina • C/Núñez de Balboa 115 • 28006 Madrid • Tfno.: (0034) 91 782 43 30/33/34 • E-mail: secretaria@ctomedicina.com • www.grupocto.es
8
1)
hematuria macro o microscópica.
Preguntas
Grupo CTO
CTO Medicina

En alrededor de un 10% de los pacientes puede aparecer


Test 2V
Distancia Pediatría
antes, abultadas, aunque no son dolorosas ni pruriginosas.
¿Cuál es su diagnóstico de sospecha?:
2) Para el estudio inicial, se recomienda la realización de una 1) Varicela.
biopsia. 2) Enfermedad de Kawasaki.
3) Las metástasis más frecuentes son pulmonares. 3) Exantema súbito.
4) Se asocia a aniridia y hemihipertrofia. 4) Púrpura de Schönlein-Henoch.
5) El estadio V es el tumor bilateral. 5) Mononucleosis infecciosa.

78. Señale la afirmación FALSA respecto a la PTI: 83. Niña de 10 meses de edad, que desde hace dos semanas pre-
1) Existe un importante descenso del número de plaquetas senta un catarro de vías altas leve con febrícula. En los últimos
circulante, aunque el número de megacariocitos en médula días la tos ha ido en aumento, presentando en la actualidad
ósea es normal. episodios de tos paroxística intensa, que acaban con un ruido
2) Cursa con importante esplenomegalia. agudo inspiratorio y, a veces, van seguidos de vómitos. En la
3) La complicación más grave de la enfermedad es la hemorragia exploración tiene 37.2oC, buen estado general, y únicamente
intracraneal. se ausculta algún roncus aislado en ambos hemitórax. En el
4) En el tratamiento se emplea gammaglobulina i.v. y los cor- hemograma destaca linfocitosis absoluta y la radiografía de
ticoides. tórax muestra infiltrados perihiliares bilaterales. ¿Cuál de
5) El tiempo de hemorragia y el tiempo de retracción del las siguientes afirmaciones es FALSA, con respecto a esta
coágulo están alterados. patología?:
1) En los lactantes de menos de 6 meses puede asociarse con
79. Niño de 2 años al que se ha realizado un análisis de sangre alta mortalidad.
por síndrome febril de 6 días de evolución, con buen estado 2) Entre las posibles complicaciones destacan la apnea, OMA,
general y sin foco infeccioso aparente. Los reactantes de fase neumonías y secuelas físicas secundarias a los esfuerzos de
aguda son normales, salvo ligera leucocitosis con predomi- la tos.
nio de linfocitos, por lo que usted tranquiliza a la madre 3) El tratamiento más adecuado es la administración de eritro-
informándole de que lo más probable es que el niño tenga micina e Igs de la tos ferina.
una infección vírica y quizá el resultado de la serología dé 4) Los adultos y adolescentes tosedores infectados son el prin-
el diagnóstico. Sin embargo en el hemograma desataca una cipal reservorio.
cifra de hemoglobina de 9,5 g/dl, con un volumen corpuscular 5) Pueden persistir alteraciones transitorias de la función pul-
medio de 70 fl. Señale la respuesta INCORRECTA acerca de monar en niños menores de 2 años.
la patología que sospecha:
1) Es la enfermedad hematológica más frecuente en la infancia. 84. Varón de 6 años que, estando previamente bien, presenta fiebre
2) Siempre hay que buscar pérdidas de sangre cuando nos de 40oC, de 24 horas de evolución, acompañada de vómitos,
planteamos esta diagnóstico. cefalea y dolor de garganta intenso. En la exploración encon-
3) Es muy frecuente en los primeros 6 meses de vida. tramos amígdalas grandes y enrojecidas, faringe hiperémica,
4) El bajo peso al nacimiento es un factor de riesgo. lengua de color rojo intenso, petequias en velo del paladar y
5) Una vez instaurado el tratamiento, el aumento de las cifras úvula, exantema rojo intenso, maculopapuloso, puntiforme,
de hemglobina tarda 2-3 semanas en aparecer. de predominio en pliegues, así como enrojecimiento difuso
en la cara, que respeta triángulo perinasobucal. Señale la
80. Respecto a los tumores de la infancia, señale la afirmación afirmación correcta:
correcta: 1) La contagiosidad remite nada más instaurar la antibioterapia.
1) La leucemia aguda es la patología oncológica más frecuente 2) El exantema afecta casi siempre a palmas y plantas.
de la infancia. 3) Puede aparecer como complicación la afectación renal,
2) El astrocitoma se localiza principalmente en fosa anterior. pero no la fiebre reumática.
3) El tumor sólido extracraneal más frecuente es el nefroblastoma. 4) El cultivo de la faringe es la prueba de laboratorio más útil
4) No se debe biopsiar un neuroblastoma por el riesgo de para establecer el diagnóstico.
sangrado. 5) El tratamiento de elección es la penicilina benzatina durante
5) Los niños con tumor de Wilms tienen un aspecto de muy 3 días.
enfermos.
85. Señale cuál de las siguientes afirmaciones es FALSA, respecto
81. Con respecto al diagnóstico del niño VIH positivo, señale cuál al sarampión:
de las siguientes afirmaciones es FALSA: 1) El período de máxima contagiosidad abarca la fase prodró-
1) En lactantes mayores de 18 meses, el diagnóstico se hace mica de la enfermedad.
igual que en adultos. 2) Son raros los casos de sarampión en lactantes menores de
2) La edad de presentación, el recuento de linfocitos CD4/CD8 y 4-6 meses.
la carga viral son factores que condicionan el pronóstico. 3) Las manchas de Koplik son patognomónicas.
3) En un niño mayor de 18 meses puede hacerse el diagnóstico de 4) Coincidiendo con la aparición del exantema, suele producirse
infección por VIH, si existen dos determinaciones positivas al una brusca elevación de la temperatura.
realizar el cultivo viral, la PCR o la determinación del Ag p24. 5) El exantema se inicia en el tronco, y desde ahí se extiende a la
4) Al nacimiento, todos los hijos de madre VIH+ tienen anti- cara y extremidades, desapareciendo en el mismo orden.
cuerpos (Acs) tipo Ig A frente al VIH.
5) Para diagnosticar la serorreversión de los Acs en un niño 86. Una de las siguientes afirmaciones NO es correcta, respecto
entre los 6 y 18 meses, son necesarios dos test de ELISA a las complicaciones del sarampión:
negativos. 1) Una de las complicaciones más frecuentes es la otitis media.
2) La neumonía de células gigantes de Hecht es más frecuente
82. Niña de 9 años, con febrícula y cansancio en los últimos 2 que la neumonía causada por sobreinfección bacteriana.
días. Hoy comienzan a aparecerle en los pies, las piernas y 3) Hasta el 50% de los niños con sarampión no complicado
alguna por los muslos, máculas rosadas que van haciéndose tienen alteraciones en el EEG, sin otros signos de alteración
más oscuras, llegando incluso a palparse las que han aparecido del SNC.

CTO Medicina • C/Núñez de Balboa 115 • 28006 Madrid • Tfno.: (0034) 91 782 43 30/33/34 • E-mail: secretaria@ctomedicina.com • www.grupocto.es
9
Pediatría
4)
Test 2V Distancia
No existe correlación entre la gravedad del sarampión y la
de la encefalomielitis aguda.
Preguntas
Grupo CTO
CTO Medicina

maculosas y rojas, que evolucionaron a vesículas claras en


tórax, cara, muslos y brazos. En la exploración destacan,
5) Puede aparecer reactivación de una tuberculosis preexistente, además, algunas costras y pústulas, 3 pequeñas úlceras en
acompañando al sarampión. la lengua y paladar, y dos lesiones similares en los labios
menores. ¿Cuál es su diagnóstico?:
87. Niña de 2 años de edad, que presenta desde ayer catarro 1) Primoinfección herpética.
de vías altas y fiebre de 38oC. Hoy acude por aparecer un 2) Síndrome pie-mano-boca.
exantema maculopapuloso rosado, no confluente, no pun- 3) Erupción variceliforme de Kaposi.
tiforme ni pruriginoso, en cara, tronco y extremidades. En 4) Varicela.
la exploración, además del exantema, destacan conjuntivas 5) Herpangina.
hiperémicas, múltiples adenopatías, dolorosas a la palpación,
en región cervical posterior y postoccipital, así como un polo 92. Un niño de 13 años presenta dolor y tumoración en la man-
de bazo. La faringe está levemente hiperémica y se observan díbula derecha, que borra ángulo y ha ido aumentando rápi-
pequeñas manchas rojas en el velo del paladar. El diagnóstico damente en pocas horas. Además, presenta fiebre de 38,5ºC
más probable es: y cefalea. Señale la afirmación cierta:
1) Sarampión. 1) El 95% de las infecciones por este paramixovirus producen
2) Rubéola. clínica.
3) Escarlatina. 2) En más del 65% de los enfermos se detecta pleocitosis en
4) Mononucleosis infecciosa. el LCR, aunque no presenten manifestaciones clínicas de
5) Exantema súbito. meningoencefalitis.
3) Con frecuencia se acompaña de pancreatitis clínica.
88. Niño de catorce meses traído a Urgencias por su madre 4) La tumefacción parotídea es indolora y existen algunos signos
al observar desconexión del medio, rigidez tónica, versión de inflamación de la piel suprayacente.
ocular y cianosis perioral, de tres minutos de duración, con 5) En un elevado porcentaje de los varones aparece esterilidad
somnolencia posterior durante cinco minutos. Al llegar a tras la orquitis postparotiditis.
Urgencias se observa temperatura de 39ºC. La exploración
neurológica es normal y la otoscopia muestra signos de otitis 93. Varón de 13 años que, desde hace una semana, presenta un
media serosa. La madre refiere que a la edad de seis meses cuadro de malestar general, fatiga y dolor abdominal. Desde
tuvo un episodio similar, también coincidiendo con fiebre. hace 24 horas presenta además odinofagia y aparición de
¿Cuál sería la actitud más correcta?: varias adenopatías cervicales posteriores. En la exploración
1) Observación durante 24 horas e iniciar tratamiento con destaca temperatura de 39oC, varias linfadenopatías de 1,5
carbamacepina. cm de diámetro en región cervical posterior, así como polo
2) Realizar TC craneal y punción lumbar. de bazo palpable, a 3 cm del reborde costal izquierdo, e
3) TC craneal, punción lumbar y hemocultivos seriados. hipertrofia amigdalar con exudado blancoamarillento. Con
4) EEG e iniciar tratamiento con ácido valproico. este cuadro, usted ya se ha hecho su impresión diagnóstica,
5) Observación, tratamiento antitérmico y seguimiento ambu- y sabe que el proceso en cuestión se caracteriza por todo lo
latorio. siguiente, EXCEPTO:
1) Su agente etiológico más probable es el virus de Epstein-
89. Señala cuál de las siguientes afirmaciones, respecto a la Barr.
rubéola y sus complicaciones, es FALSA: 2) Se trata de un virus linfotropo, que afecta a los linfocitos de
1) La artritis suele afectar a mujeres, grandes articulaciones y estirpe T.
suele resolverse sin dejar secuelas. 3) Aunque la hepatomegalia sea poco frecuente, sabe que
2) Pueden aparecer complicaciones neurológicas, siendo éstas hasta en un 50% es posible detectar elevación de las tran-
mucho menos frecuentes que en el caso del sarampión. saminasas.
3) La profilaxis con gammaglobulina está indicada en mujeres 4) Dentro de la evolución de la enfermedad, hasta en un 80%
embarazadas durante el primer trimestre de la gestación. puede aparecer un exantema maculopapuloso al administrar
4) En casos de rubéola congénita, el paciente puede excretar ampicilina.
virus durante meses. 5) Para el diagnóstico, es útil la determinación de Acs heterófilos.
5) La relación entre formas subclínicas/enfermedad patente es
de 2/1. 94. ¿Cuál de las siguientes es una característica propia del SIDA
infantil respecto al SIDA en los adultos?
90. Paciente de 17 meses de edad, que acude a urgencias por 1) Mayor tiempo de incubación.
convulsión tonicoclónica generalizada y fiebre de 39oC. En 2) Hipergammaglobulinemia tardía.
la analítica destaca leucocitosis y neutrofilia. Se ingresa para 3) Menor número de infecciones bacterianas.
observación. A los 3 días desaparece la fiebre, apareciendo 4) Mayor linfopenia.
un exantema. Señale la opción verdadera, con respecto al 5) Menor número de infecciones oportunistas.
proceso que padece este paciente:
1) Si al tercer día de evolución desaparece la fiebre y aparece 95. Respecto al Síndrome de Muerte Súbita Infantil o del Lactante
un exantema vesiculoso en tronco y miembros inferiores, (SMSL), señale cuál de los siguientes NO constituye un factor de
podemos establecer el diagnóstico de roséola. riesgo:
2) La lengua en fresa roja es un hallazgo frecuente en esta 1) Antecedentes familiares.
patología. 2) Prematuridad.
3) El agente etiológico más frecuente es el herpesvirus humano 3) Posición para dormir en decúbito supino.
tipo 6. 4) El sexo masculino.
4) Es muy frecuente la alteración del LCR. 5) El tabaquismo materno.
5) El tratamiento de elección es penicilina vo.
96. En la tetralogía de Fallot, NO es cierto que:
91. Una niña de 3 años, previamente sana, bien vacunada, co- 1) La clínica depende del grado de obstrucción a la salida del
menzó hace 3 días con unas lesiones cutáneas pequeñas, ventrículo derecho.

CTO Medicina • C/Núñez de Balboa 115 • 28006 Madrid • Tfno.: (0034) 91 782 43 30/33/34 • E-mail: secretaria@ctomedicina.com • www.grupocto.es
10
2)
Preguntas
Grupo CTO
CTO Medicina
Test 2V
Aparece cianosis progresiva y disnea de esfuerzo, que mejora
en posición de cuclillas.
Distancia Pediatría

3) En la Rx de tórax aparece una cardiomegalia con pedículo


cardíaco estrecho y plétora pulmonar.
4) En la auscultación cardíaca se detecta un soplo sistólico en
borde esternal izquierdo.
5) En el ECG aparece hipertrofia de ventrículo derecho.

97. Señale cuál de los siguientes signos radiológicos es típico de


la coartación aórtica:
1) Silueta cardíaca en zueco.
2) Muescas o escotaduras costales.
3) Signo del hombre de nieve.
4) Silueta cardíaca estrecha u ovalada.
5) Síndrome de la cimitarra.

98. A la guardia llega un lactante de 15 meses con una convulsión


tonicoclónica generalizada. Sobre la actitud que Vd. adoptaría,
señale la respuesta INCORRECTA:
1) Lo más frecuente es que se trate de una convulsión febril, por
lo que se debe establecer la temperatura del paciente.
2) Lo habitual es que la duración sea inferior a 10 minutos.
3) Si el inicio es focal, se debe realizar un EEG.
4) Hasta un 95% de los casos presentan recurrencias posteriores,
que suelen ceder hacia los 5 años.
5) El tratamiento profiláctico con diacepam rectal sólo se
instaura ante convulsiones febriles muy frecuentes.

99. Varón de 6 meses de edad, con movimientos de flexión brus-


ca de cuello, brazos y piernas hacia el tronco, con posterior
extensión de miembros, que aparecen al despertar. Entre sus
antecedentes, no destaca ningún factor predisponente. La
exploración neurológica y el TC craneal son normales y en el
EEG se observa un patrón de hipsarritmia. Usted sabe que el
tratamiento de primera elección, de entre los siguientes, es:
1) Valproato sódico.
2) Fenobarbital.
3) ACTH.
4) Paraldehído.
5) Etosuximida.

100. Acude a nuestra consulta un niño de 8 años, sobreactivo sin


una finalidad, tanto en casa como en sus actividades escola-
res, que además es impulsivo, agresivo con sus compañeros,
tiene crisis de frustración con facilidad y dificultades en su
rendimiento escolar. ¿Cuál de las siguientes respuestas consi-
dera FALSA, respecto al cuadro que parece presentar nuestro
paciente?:
1) Se considera que puede existir una predisposición genética.
2) Se ha relacionado con alteración en receptores dopami-
nérgicos.
3) Es un trastorno más frecuente en varones.
4) Se han demostrado alteraciones en estudios de neuroimagen
y de metabolismo cerebral.
5) El tratamiento se basa en fármacos sedantes y apoyo psico-
pedagógico.

CTO Medicina • C/Núñez de Balboa 115 • 28006 Madrid • Tfno.: (0034) 91 782 43 30/33/34 • E-mail: secretaria@ctomedicina.com • www.grupocto.es
11
Distancia

Test 2VPediatría
Grupo CTO
CTO Medicina

Comentarios

Pregunta 1.- R: 4 el lado derecho. Respecto a la patología que sospecha, señale la


• Los niños duplican su peso a los 5 meses de vida, lo triplican al año afirmación FALSA:
y lo cuadruplican a los dos años. Duplican la talla del nacimiento 1) Se produce por alteración en el cierre de los canales pleurope-
con 4 años. ritoneales posterolaterales.
• El cierre de la fontanela anterior ocurre entre los 9 y los 18 meses 2) El diagnóstico suele hacerse por la radiografía, apreciándose
de vida, y la posterior se cierra antes (entre las semanas 6 y 8). asas intestinales en el tórax y desplazamiento del mediastino.
• El perímetro torácico en el momento del nacimiento es menor 3) El cierre del defecto anatómico es considerado una urgencia
que el del perímetro cefálico, y ambos se igualan alrededor del quirúrgica.
año de vida. 4) Con frecuencia se asocia a otras malformaciones congénitas
• La piel del pretérmino es fina, friable, hace equimosis con facilidad, importantes.
y presenta lanugo (vello fino y evanescente). El vérnix caseoso, 5) Entre las potenciales complicaciones graves destaca el desarrollo
unto sebáceo de la piel, está presente en los RN a término. Los de hipertensión pulmonar y la hipoplasia pulmonar.
RN postérmino tienen una piel quebradiza y apergaminada, hecho
que les confiere un aspecto “de viejillo”. Los quistes de millium, el La hernia diafragmática congénita posterior (de Bochdaleck)
eritema tóxico, la melanosis pustulosa y los angiomas planos son es un cuadro severo que aboca a la instauración de hipertensión
lesiones sin significado patológico. pulmonar, uno de los más graves procesos que le pueden ocurrir
a un RN.

• Acondroplasia. A través de este defecto, vísceras abdominales migran hacia el


• Síndrome de Apert. • Osteogénesis imperfecta. tórax, con lo cual el abdomen aparece excavado (vacío de contenido)
• Hipotiroidismo atiretósico. • Premadurez. y hay un compromiso respiratorio severo secundario a la compresión
• Disostosis cleidocraneal. • Picnodisóstosis.
pulmonar (distrés, cianosis).
• Síndrome de Hallerman- • Síndrome rubeólico.
Si hacemos una Rx de tórax, advertiremos la presencia de asas
Streiff. • Síndrome de Russell-Silver. intestinales en tórax, y el mediastino estará desplazado por el efecto
• Hidrocefalia. • Trisomías 13, 18, 21. masa de éstas.
• Hipofosfatasia. • Raquitismo por deficiencia
• Retraso en el crecimiento de vitamina D. El cierre quirúrgico se hará con celeridad, pero antes, para evitar
intrauterino la hipertensión pulmonar, durante 10-12 horas forzaremos una
Pregunta 1. Trastornos relacionados con la fontanela anterior grande situación de alcalosis (el pH alcalino vasodilata las arterias pulmo-
nares) para minimizar la hipertensión pulmonar.La hernia diafrag-
mática congénita se asocia a otras malformaciones, como malrotación
intestinal, cardiopatías, malformaciones vasculares...
Pregunta 2.- R: 5
• En un RN normal, al poco de nacer se cierran los cortocircuitos de
Pregunta 4.- R: 4
la circulación fetal: ductus arterioso, conducto venoso de Arancio
No olvides repasar el test de Apgar antes del examen MIR:
y foramen oval.
• El meconio se elimina en las primeras 48 horas de vida, y la primera
micción suele ocurrir en las primeras 24 horas.
• En el RN, tanto la capacidad de filtración como la de concentración Parámetros 0 1 2
de la orina están mermadas. Esfuerzo
• La frecuencia cardíaca normal en un RN oscila entre 90-180 (se respiratorio
Ausente Débil Llanto
consideraría taquicardia en un adulto) y la FR, es de unas 30-40
rpm (se consideraría taquipnea en un adulto). FC Ausente <100 lpm >100 lpm
• Los RN presentan leucocitosis fisiológica (>11000 leucocitos/mm3), Movimientos
y nacen con una Hb alta (16-18 g/dl). Tono muscular Hipotonía Ligera flexión
activos
Pregunta 3.- R: 3 Irritabilidad
No Muecas Tos
Recién nacido a término que presenta a las pocas horas de vida refleja
cianosis e intenso distress respiratorio. A la exploración, usted Color Palidez Acrocianosis Rosado
observa un abdomen excavado y ausculta el latido cardíaco en

CTO Medicina • C/Núñez de Balboa 115 • 28006 Madrid • Tfno.: (0034) 91 782 43 30/33/34 • E-mail: secretaria@ctomedicina.com • www.grupocto.es
1
Pediatría
Pregunta 5.- R: 3
Test 2V Distancia

Señale la afirmación correcta respecto al test de Apgar:


Comentarios
Grupo CTO
CTO Medicina

Pregunta 8.- R: 3
El enunciado de esta pregunta muestra:
1) Se debe practicar únicamente a los RN patológicos o supues- • Un RN macrosómico.
tamente patológicos. • Nacido de nalgas.
2) Entre los parámetros que se valoran se encuentra la frecuencia • En el contexto de un parto laborioso y traumático (fractura de
respiratoria. clavícula, parálisis braquial inferior, caput grande).
3) Se debe realizar al minuto y a los 5 minutos de vida. • Y que muestra signos clínicos de shock (polipnea, hipotensión).
4) Un test de Apgar de 4 al minuto de vida implica un alto riesgo
de parálisis cerebral. Con estas características (parto traumático, presentación podá-
5) La palidez cutánea generalizada supone una puntuación de 1 lica, macrosomía, shock), rápidamente has de pensar en la posi-
en el test de Apgar. bilidad de una hemorragia suprarrenal. A su vez ésta se expresa
como:
El test de Apgar se debe realizar a todo recién nacido, independien- • Masa palpable en uno de los flancos, en la exploración (opción 1).
temente de su edad gestacional, al minuto, 5 minutos y 10 minutos • Anemización, como cualquier hemorragia (opción 2), con aumento
de vida (respuesta 3 correcta). de la oferta de bilirrubina indirecta e ictericia (opción 4).
Entre los parámetros que valora se encuentran: color, frecuencia • En el plano analítico, cabe reseñar la aparición de hipoglucemia
cardíaca, respuesta a la estimulación, tono muscular y esfuerzo res- mantenida (lo más típico), y una situación de hipoaldosteronis-
piratorio (NO frecuencia respiratoria). mo, con sodio bajo y potasio alto (opción 5).
A pesar de una puntuación baja en los primeros minutos de vida,
no sirve para establecer valoraciones pronósticas. Una puntuación La opción 3 es falsa. No tiene por qué haber desviación izquierda
inferior a 3, mantenida más allá de los 20 minutos de vida, sí puede en el hemograma. Este hecho sería típico de la sepsis neonatal.
predecir una elevada morbimortalidad.
La palidez cutánea generalizada supone una puntuación de 0 Pregunta 9.- R: 5
en el test de Apgar (ver el cuadro del comentario de la pregunta RNPT de 3 días de vida y 2300 g de peso, que presenta en la
anterior). exploración una tumoración abdominal cubierta por piel, en la
base del cordón umbilical, fácilmente reductible, palpándose en-
Pregunta 6.- R: 3 tonces un orificio herniario rodeado de bordes duros. ¿Cuál sería
Debes conocer el cefalohematoma y sus diferencias con respecto el diagnóstico más probable?:
al caput succedaneum. Esta pregunta te va a servir para recordar las 1) Onfalocele.
principales características del cefalohematoma: 2) Granuloma umbilical.
• Suele afectar a uno o ambos parietales. 3) Gastrosquisis.
• Es una hemorragia subperióstica. A veces puede producir, si es 4) Persistencia del uraco.
de suficiente cuantía, ictericia. Esta ictericia, a su vez, puede 5) Hernia umbilical.
demandar fototerapia para su resolución.
• El cefalohematoma se distribuye respetando suturas (queda Una tumoración abdominal, fácilmente reductible, en la base del
confinada a un hueso en concreto), no como el caput, que cordón umbilical y cubierta de piel, coincide con la descripción de
las sobrepasa (aparece en el área suprayacente a varios una hernia umbilical (respuesta 5 correcta). Si la tumoración estuviera
huesos). recubierta por peritoneo se trataría de un onfalocele y, si no tiene
• La piel suprayacente está indemne en el cefalohematoma, y cubierta externa, sería una gastrosquisis.
presenta tumefacción, equimosis y petequias en el caput. El granuloma se trata de un tejido blando, granular y de color
• El cefalohematoma tarda más en visualizarse que el caput, y no rojizo o rosado, que puede tener una secreción mucopurulenta. La
aparece hasta varias horas después del parto. persistencia de uraco se manifestaría como una fístula que exuda un
líquido ácido.
Pregunta 7.- R: 2
Recién nacido mediante un parto de nalgas presenta la mano Pregunta 10.- R: 3
derecha caída. Los dedos de dicha mano aparecen semiabiertos El enunciado de esta pregunta muestra:
y no es capaz de agarrar el dedo de su madre con esa mano pero • A un RN casi a término (36 semanas).
sí con la izquierda. Señale la respuesta incorrecta respecto a la • Con signos de dificultad respiratoria (retracción subesternal,
lesión obstétrica que sufre este recién nacido: subcostal y quejido) y polipnea.
1) Se debe a una lesión en las raíces inferiores del plexo braquial. • De aparición en las primeras dos horas de vida.
2) El reflejo de Moro no estará presente en el lado derecho. • Que evoluciona favorablemente con la administración de oxí-
3) El parto de nalgas favorece esta situación. geno.
4) Puede asociarse con síndrome de Horner.
5) Este tipo de parálisis es menos frecuente que la parálisis de Erb. Recuerda este “retrato robot” para el MIR, pues corresponde a un
distrés respiratorio tipo II o taquipnea transitoria del RN o síndro-
Caso clínico de una parálisis braquial inferior, también conocida me de Avery. Esta variante de dificultad respiratoria presenta:
como parálisis de Klumpke. Se debe a la afectación de las raíces C7 • Como desencadenante, un parto por cesárea o un parto vagi-
y C8 del plexo braquial. El recién nacido presenta la mano caída con nal rápido, con la incapacidad secundaria para llevar a cabo
el reflejo de prensión palmar del lado afecto ausente. Sin embargo, una reabsorción correcta del líquido que anega los pulmones
el reflejo de Moro estará presente ya que las raíces superiores no se intraútero.
han afectado (respuesta 2 incorrecta). Si se lesiona también la primera • Un desenlace rápido, o mejoría sintomática en los primeros 1-2
raíz torácica aparecerá síndrome de Horner asociado. días de vida.
• El patrón radiológico clásico consiste en la aparición de pulmo-
nes levemente congestivos, líquido en cisuras, derrame pleural
PARÁLISIS de pequeño tamaño.
P. ERB-DUCHENNE P. KLUMPKE
BRAQUIAL
Pregunta 11.- R: 3
Raíces (C4) - C5 - C6 C7 - C8 - (T1) La enfermedad de membrana hialina se caracteriza por:
Brazo en adducción • Dificultad respiratoria que afecta a un RN pretérmino, que pro-
Clínica Mano caída gresa en las primeras horas de vida, y que se muestra refractaria
y rotación interna
a la aplicación de oxígeno indirecto.
No presente o • Se debe a un déficit de surfactante (su tratamiento es, precisa-
R. Moro Presente
asimétrico mente, la administración de éste), y a una inmadurez pulmonar,
R. Prensión palmar Presente No presente con cociente L/E<2.
• A nivel analítico destaca una insuficiencia respiratoria mixta,
Asociaciones C4 - Parálisis frénica T1 - S. Horner
con hipoxia franca, llamativa, importante.
Pregunta 7. Parálisis braquial • A nivel radiológico, los hallazgos clásicos son: colapso pulmonar,
patrón reticulogranular con broncograma aéreo de predominio

CTO Medicina • C/Núñez de Balboa 115 • 28006 Madrid • Tfno.: (0034) 91 782 43 30/33/34 • E-mail: secretaria@ctomedicina.com • www.grupocto.es
2
Comentarios
Grupo CTO
CTO Medicina

basal, rico en atelectasias, con aspecto global de vidrio esme-


rilado. Su grado extremo es el “pulmón blanco”, totalmente
Test 2V Distancia Pediatría
hialina, llamaremos a la demanda de oxígeno síndrome de Wilson-
Mikity. La expresión radiológica de la DBP comprende la aparición de
atelectásico. condensaciones parcheadas, tractos fibrosos y áreas de hiperclaridad,
hecho que confiere al pulmón un aspecto de esponja.
La DBP provoca una elevación de las resistencias pulmonares; por
tanto, fomenta a aparición de una hipertensión pulmonar secundaria
que sobrecarga el ventrículo derecho de forma crónica, abocando éste
a un fracaso potencial del mismo.
En el tratamiento de la DBP resulta esencial tanto la administración
de oxígeno como la restricción de líquidos, para minimizar las sobre-
carga pulmonar y cardíaca. Esta restricción ha de ser complementada
con la administración de diuréticos. En algunos casos se recurre,
además, a los broncodilatadores.

Pregunta 15.- R: 4
Un recién nacido presenta taquipnea y en la radiografía de
tórax se observan imágenes vasculares pulmonares prominentes,
diafragma aplanado y líquido en las cisuras. No existe hipoxemia,
hipercapnia o acidosis. ¿Cuál de las siguientes enfermedades es
más probable?:
1) Enfermedad de la membrana hialina.
2) Aspiración de meconio.
3) Neumomediastino.
4) Taquipnea transitoria del recién nacido.
5) Síndrome de Wilson-Mikity.

La analítica normal con la radiología descrita debe hacernos pensar


en la taquipnea transitoria del RN (respuesta 4 correcta). El líquido en las
cisuras traduce el retraso en la absorción del mismo. También aparecen
marcas vasculares prominentes junto con discreta hiperinsuflación que
hace que el diafragma se aplane. A diferencia del resto de opciones, el
Pregunta 11. Factores que contribuyen en la patogenia distress respiratorio en la taquipnea transitoria del RN suele ser leve-
de la EMH. "Círculo vicioso". moderado, con poca o ninguna alteración gasométrica.
Pregunta 12.- R: 4 Pregunta 16.- R: 5
Una de las complicaciones de la enfermedad de membrana hialina Uno de cada doscientos RN alimentados al pecho presenta un
es la aparición de un ductus arterioso persistente. El “retrato robot” cuadro que se conoce como síndrome de Arias, y que consiste en
de DAP puede quedar delimitado con lo que aparece en esta pre- la aparición de ictericia. Ésta se suele extender entre los días 7 y 30
gunta: de vida. Generalmente, no reviste trascendencia (niveles máximos de
• Pretérmino. bilirrubina: 20-30 mg/dl). Por tanto, la actitud del pediatra ha de ser
• Con enfermedad de membrana hialina. tranquilizadora. No está indicado retirar la lactancia materna (opción
• Que, después de la fase de mejoría sintomática de la misma 5, verdadera). En el caso de nuestro paciente (bilirrubina menor de
(3-5 días de vida). 20 mg/dl), tampoco está indicada ninguna otra maniobra adicional.
• Se deteriora bruscamente, con nueva aparición de distrés res-
piratorio.
• Retención de CO2 en el plano analítico. RN atérmino RN pretérmino
• Aparición de soplo continuo “en maquinaria” o de Gibson,
• Y pulsos exaltados, muy llamativos. Inicio 2-3 días 3-4 días
Duración 5-7 días 6-8 días
Al no ser una cardiopatía cianosante, las opciones 1 y 3 quedan
descartadas. La coartación de aorta cursa con pulsos débiles. La CIV Concentración 12 mg/dl a los 3-4 14 mg/dl a los 6-8
cursa sin cianosis, pero el soplo aparece desde las primeras horas de máxima días días
vida, y es pansistólico (no continuo). Pregunta 16. Ictericia fisiológica en neonatos

Pregunta 13.- R: 4
El enunciado de la pregunta describe a la perfección el síndrome Pregunta 17.- R: 2
de aspiración meconial (SAM): El RNPT, sobre todo si sufre patología importante, es especialmente
• Un RN postérmino (44 semanas de edad gestacional). vulnerable a la aparición de toxicidad neurológica por bilirrubina
• Con antecedentes de sufrimiento fetal (recuerda la secuencia (kernicterus). Es conveniente memorizar las cifras por encima de las
SFA...hipoxia...contracción intestinal con relajación del esfínter cuales hay que exanguinotransfundir:
anal...eliminación de meconio). • En RNPT: por encima de 17 mg/dl de bilirrubina.
• Que presenta distrés respiratorio de inicio precoz. • En RNT, por encima de 25 mg/dl.
• Con hiperinsuflación torácica (el meconio provoca atrapamiento
aéreo por un mecanismo valvular, que permite la entrada de Nuestro neonato presenta 20 mg/dl de bilirrubina total e importante
aire pero limita su salida). patología de base; en situaciones como esta debemos ser agresivos, y
• Con tendencia, debido al celo de guardar aire, a un rebosamien- proceder a la exanguinotransfusión (opción 2, correcta).
to del alvéolo, con posibilidad de ruptura del mismo, y aparición
de aire ectópico en sus distintas modalidades (en el enunciado Pregunta 18.- R: 2
aparece un neumotórax: hiperresonancia pulmonar, abolición Una ictericia precoz, de aparición en las primeras 24-48 horas de
del murmullo vesicular, dolor con inquietud, apnea). vida, ha de obligarnos a pensar en tres grandes patologías:
• Recuerda la radiología típica: infiltrados algodonosos, pulmones • Hemorragia aguda en el contexto de un parto traumático.
hiperinsuflados, áreas ampollosas y aparición de aire ectópico. • Isoinmunización (test de Coombs directo positivo, anemización).
• Infecciones (sepsis neonatal, infecciones connatales).
Pregunta 14.- R: 2
El equivalente de la EPOC del adulto en los neonatos que sufrieron Hay otras ictericias de aparición más perezosa, (nunca en las
una enfermedad de membrana hialina es la displasia broncopulmo- primeras horas de vida) y que se perpetúan más allá del mes de vida:
nar, que se define como la necesidad de oxígeno a los 28 días de vida hipotiroidismo, galactosemia, ictericia obstructiva, Gilbert, metabo-
(o a las 36 semanas de edad corregida). Si no hay antecedentes de lopatías, etc.

CTO Medicina • C/Núñez de Balboa 115 • 28006 Madrid • Tfno.: (0034) 91 782 43 30/33/34 • E-mail: secretaria@ctomedicina.com • www.grupocto.es
3
Pediatría

DIAGNÓSTICO
Test 2V
Tipo de BR
Distancia

Ictericia
Comentarios
Grupo CTO
CTO Medicina

Concentración
Observaciones
APARECE DESAPARECE máxima de BR
Ictericia fisiológica
Relación con el
RNT Indirecta 2-3 días 4-5 días 2-3 días
grado de madurez
RNPT Indirecta 3-4 días 7-9 días 6-8 días
Hipoxia, dificultad respirato-
ria, déficit carbohidratos.
HiperBR 2ª a factores
Influencias hormonales:
metabólicos
cretinismo, hormonas.
RNT Indirecta 2-3 días Variable 1ª semana
Fact. Genéticos: Crigler-
RNPT Indirecta 3-4 días Variable 1ª semana
Najjar, hiperBR familiar
transitoria.
Eritroblastosis, hemólisis con-
Cuadros hemolíticos y
Indirecta Primeras 24 horas Variable Variable génita, picnocitosis infantil,
hematomas
vit. K, hematomas.
Sepsis bacteriana, pielonefri-
Factores hemolíticos y Indirecta y tis, hepatitis, toxoplasmosis,
Primeras 24 horas Variable Variable
hepatotóxicos mixtos directa enf. inclusiones citomegáli-
cas, rubeola
Indirecta y Atresia biliar, galactosemia,
Lesión hepatocelular 2-3 días Variable Variable
directa hepatitis e infecciones.
Pregunta 18. Rasgos diagnósticos para los diferentes tipos de ictericia neonatal

Pregunta 19.- R: 5 En la Rx simple de abdomen encontraremos:


La patología digestiva más típica de un RNPT (como el de nuestro • Neumatosis intestinal (signo diagnóstico de la NEC)
enunciado, que tiene una edad gestacional menor de 37 semanas) • Edema de asas.
es la enterocolitis necrotizante, cuadro caracterizado por vómitos, • Patrón en miga de pan.
distensión abdominal y deposiciones hemorrágicas. Puede ser el pro- • Asa fija.
legómeno de una sepsis de origen entérico. A nivel analítico, destaca • Gas en la vena porta.
la aparición de neutropenia y trombopenia. Estudios experimentales • Neumoperitoneo;si existe perforación.
recientes avalan que se puede prevenir si se utiliza lactancia materna.
Para su tratamiento, hay que imponer reposo gástrico (hay que nutrir Manejo:
por vía parenteral) y emplear antibióticos. En casos refractarios al tra- • Dieta absoluta, fluidoterapia y descompresión nasogástrica.
tamiento médico, o si se perfora el segmento intestinal afecto (visible • Antibióticos que cubran gérmenes anaerobios y gramnegati-
a través de neumoperitoneo), se recurrirá a la cirugía. Por tanto, la vos.
opción 5 es correcta; la opción 1 sería insuficiente. En la fase aguda, la • Tratamiento quirúrgico en caso de perforación intestinal o sepsis
realización de un enema opaco va a complicar la situación de nuestro refractaria al tratamiento médico.
paciente (opción 4, falsa).
En este caso, la presencia de gas en la pared intestinal (neumatosis
Pregunta 20.- R: 3 intestinal) no es indicación de cirugía urgente (respuesta incorrecta
Un RN pretérmino, con antecedentes de sufrimiento fetal, 3). Sí lo sería la presencia de neumoperitoneo.
comienza con distensión abdominal importante, vómitos y depo-
siciones hemorrágicas. En la Rx de abdomen se observa edema de Pregunta 21.- R: 2
asas intestinales, con un patrón en “miga de pan” y presencia de Los niveles séricos de glucosa disminuyen en las primeras 3 horas
gas en la pared intestinal. Respecto a esta patología, señale cuál de vida, aumentando posteriormente en los recién nacidos sanos.
de las siguientes afirmaciones es FALSA: A menudo la hipoglucemia es asintomática, especialmente en los
1) Las situaciones de hipoxia y bajo gasto predisponen a esta hijos de madre diabética. Los prematuros tienen síntomas con más
patología. frecuencia que otros grupos de pacientes.
2) La primera manifestación clínica es la aparición de distensión
abdominal. Estas manifestaciones clínicas son variables y pueden confundirse
3) La presencia de neumatosis intestinal es indicación de cirugía con otras patologías: temblores, apatía, cianosis episódica, convulsio-
urgente. nes, crisis de apnea o por el contrario taquipnea, llanto, hipotonía,
4) La supresión de la alimentación oral y el reposo intestinal con letargia, rechazo del alimento y movimientos oculares anómalos, por
descompresión mediante SNG es necesario. orden de frecuencia. Estas manifestaciones pueden aparecer desde
5) Se han involucrado en su etiología a diversos patógenos como pocas horas tras el parto, hasta una semana después.
E.coli, C.perfringens, Rotavirus y S. epidermidis.
La incidencia disminuye con la alimentación precoz y aumenta en
La enterocolitis necrotizante (NEC) es una lesión isquémico- los hijos de madre diabética, con las perfusiones de glucosa maternas
necrótica que suele afectar a íleon distal y colon proximal, y durante el parto, la premadurez, el crecimiento intrauterino retardado,
que se sigue habitualmente de sepsis bacteriana a partir de foco la hipoxia y la hipotermia.
digestivo.
Las respuestas propuestas, excepto la 2, que es una de las manifes-
Se consideran causas que predisponen a la isquemia: la polici- taciones, no de las causas de hipoglucemia neonatal, son ejemplos de
temia, el inicio de la alimentación muy pronto y con elevados volú- los 4 grupos fisiopatológicos por los que se puede dar esta situación:
menes y concentraciones y situaciones de hipoxia y de bajo gasto. hiperinsulinismo (hijo de madre diabética, eritroblastosis fetal grave,
También se han involucrado a diversos patógenos como E. coli, C. síndrome de Beckwith, hiperinsulinismos familiares), disminución
perfringens, Rotavirus y S. epidermidis, pero en la mayoría de los de las reservas hepáticas de glucógeno y grasas corporales por mala
casos no se detecta ningún germen responsable. La lactancia materna nutrición intrauterina (algunos prematuros, CIR), aumento despropor-
es un factor protector. cionado de las necesidades metabólicas (enfermedades graves como
cuadros de dificultad respiratoria importante, insuficiencia cardíaca,
Cursa con distensión abdominal (que sería el primer signo) y de- enterocolitis necrotizante,...), enfermedades metabólicas congénitas
posiciones sanguinolentas en la 2ª semana de vida. Con frecuencia (glucogenosis, alteraciones del ciclo de la urea, alteraciones de la
es un cuadro de inicio insidioso, que acaba dando lugar a una sepsis, oxidación de los ácidos grasos, galactosemia, intolerancia a la fructosa,
pudiendo acabar en shock y muerte. las principales).

CTO Medicina • C/Núñez de Balboa 115 • 28006 Madrid • Tfno.: (0034) 91 782 43 30/33/34 • E-mail: secretaria@ctomedicina.com • www.grupocto.es
4
Pregunta 22.- R: 1
Comentarios
Grupo CTO
CTO Medicina

Los hijos de madre consumidora de opiáceos no tienen más


Test 2V Distancia
Pregunta 26.- R: 4
Pediatría

¿Cuál sería la pauta más correcta a llevar a cabo en el RN


riesgo de desarrollar malformaciones congénitas (opción 1, falsa). de madre con la siguiente serología: Ag s + frente al virus de la
Recuerda que no ocurre lo mismo en los hijos de cocainómanas, en hepatitis B?:
los cuales hay mayor probabilidad de desarrollar hemorragias cere- 1) Vacuna.
brales y patología ocular. 2) Gammaglobulina.
El síndrome de abstinencia a heroína aparece más precozmente 3) Interferón Alfa.
que el de metadona (opción 2: correcta), es menos convulsivógeno 4) Vacuna + Gammablobulina.
(opción 3: correcta), y desaparece antes. 5) Vacuna+ gammaglobulina+ interferón.
Los opiáceos protegen frente a la EMH (opción 5: correcta), y su
abstinencia, en el período neonatal se trata con morfina oral (opción Los hijos de madres con HBsAg positivo, deben recibir gamma-
4, correcta). globulina contra hepatitis B durante las 12 primeras horas de vida.
A su vez, se les debe administrar junto con ella la primera dosis de
Pregunta 23.- R: 3 vacuna de VHB.
Con respecto al hijo de madre diabética, NO es cierto que:
1) Puede expresarse clínicamente retraso de la evacuación de Pregunta 27.- R: 4
meconio. La opción 1 es correcta, pues en el defecto global de hormonas
2) Si la madre sufre trastornos vasculares, el niño puede ser CIR. hipofisarias habrá un déficit de LH, con lo cual secundariamente habrá
3) El 75% de los hijos de madre diabética desarrollarán después un déficit de testosterona, que motivará la aparición de un micropene;
hipoglucemia sintomática. a su vez, los déficits de GH y ACTH asocian hipoglucemia.
4) Tienen mayor incidencia de distress respiratorio. La opción 2 es también correcta: los niveles bajos de PTH cursan
5) La hipoplasia de colon izquierdo es una malformación intestinal con hipocalcemia, que puede derivar en tetania (por ejemplo, en el
a tener en cuenta en estos niños. seno de un síndrome de DiGeorge).
La opción 3 es, a su vez, correcta. La hemorragia suprarrenal suele
Problemas observados frecuentemente en HMD: aparecer en el marco de un parto traumático de nalgas. También hay
• Mayor mortalidad fetal y neonatal. casos de hemorragia suprarrenal fulminante surgidos en el contexto
• Polihidramnios. de una sepsis (síndrome de Waterhouse-Friederichsen). Indepen-
• Macrosomía con visceromegalia, si la madre no tiene vasculo- dientemente de la causa que la origine, la insuficiencia suprarrenal
patía; si la tiene, CIR. cursa con: hipoglucemia mantenida, vómitos, diarrea, deshidratación,
• Estenosis subaórtica, con hipertrofia septal asimétrica. hiponatremia e hiperpotasemia (opción 5, correcta).
• EMH, por disminución de la síntesis de surfactante. La opción 4 es falsa, pues el crecimiento intrauterino es indepen-
• Alteraciones metabólicas: hipoglucemia, con frecuencia asin- diente de GH; por este motivo, los RN con déficit de GH no tienen
tomática (respuesta 3 incorrecta). por qué nacer con peso y talla bajos.
• Policitemia y sus consecuencias.
• Mayor incidencia de malformaciones congénitas: Pregunta 28.- R: 1
- Cardíacas (malformaciones más frecuentes en el HMD). La enfermedad hemorrágica del RN es una grave entidad pa-
- Colon izquierdo hipoplásico (malformación digestiva más togénica, debida a un déficit de factores de coagulación vitamina
frecuente). K-dependientes. Muchas veces surge en hijos de madres tratadas con
- Agenesia lumbosacra (malformación más característica). fenobarbital o fenitoína, o en niños portadores de coagulopatía. La
enfermedad de presentación tardía puede obedecer a malabsorción
Pregunta 24.- R: 3 o a defectos hepáticos. Por esta razón, está indicado hacer un estudio
Los factores de riesgo para padecer una sepsis por C. Albicans son: completo de coagulación. Su expresividad clínica es muy variada,
presencia de catéteres, nutrición parenteral prolongada, tratamiento desde hemorragia digestiva hasta hemorragia intracraneal, pasando
con antibióticos, hospitalización prolongada, colonización cutánea por un abrupto cuadro de CID.
por Candida. No es más frecuente en el hijo de madre diabética (ojo: La opción 1 es incorrecta. Para tratar la enfermedad hemorrágica
la Candida es más frecuente en diabéticos de todas las edades, pero del RN, hay que emplear vitamina K por vía IV (la dosis profiláctica
no por ello en sus hijos). de 1 mg i.m. no es suficiente) y plasma fresco congelado.
Los gérmenes más frecuentemente implicados en las sepsis tardías
son: en las no nosocomiales, Streptococcus agalactiae, E. coli y Listeria; Pregunta 29.- R: 5
dentro de las de ámbito nosocomial, destacan S aureus y P. aeruginosa. Recuerda que el valor de la Hb en un RN es mayor que en el adulto
Los serotipos con factores de adhesión a las meninges de Streptococcus (memoriza como media 16 g/dl).
agalactiae (serotipo III) y E. coli (serotipo K1) son los más frecuentemen- Una de las causas más frecuentes de anemia en el RN es la anemia
te implicados en las meningitis neonatales. El Streptococcus agalactiae, hemolítica por isoinmunización anti-ABO (suele ser menos grave) y
seguido de E. coli es la bacteria más frecuentemente implicada en anti-D (suele ser más grave).
la sepsis neonatal de inicio precoz. El antibiótico más eficaz frente La detección de hematíes fetales en sangre materna se conoce
a Listeria es la ampicilina; no ha de faltar por ello en el tratamiento como test de Kleihauer-Betke, y sirve para valorar transfusiones feto-
empírico de toda sepsis neonatal precoz. maternas ocurridas intraútero.
En fases iniciales de una hemorragia aguda, al igual que en cual-
Pregunta 25.- R: 1 quier paciente (independientemente de su edad), la hemoglobina
Para diagnosticar una sepsis neonatal has de saber que: no se altera, y hemos de fiarnos del contexto clínico para valorar la
• Cualquier neonato, severidad de la misma.
• Con factores de riesgo (fiebre materna intraparto, datos de La hemoglobina comienza a descender a las 48 horas de vida; el
corioamnionitis, bolsa rota prolongada, madre portadora de descenso alcanza un valor máximo antes en un RNPT (6 semanas)
estreptococo grupo B), que en los RNT (8 semanas). Esto es así porque los RNPT tienen
• Y clínica inespecífica (palidez, letargia, llanto agudo, inesta- menor disponibilidad de hierro para hacer la eritropoyesis, y por-
bilidad vasomotora, rechazo de las tomas, taquipnea, distrés que presentan menos sensibilidad a la EPO. Por tanto, la opción
respiratorio), 5 es falsa.
• Junto con hemograma que muestre leucopenia (o leucocitosis
extrema), neutropenia o desviación izquierda, Pregunta 30.- R: 4
• La puede padecer. El hijo de esta pareja puede desarrollar tanto una isoinmuniza-
ción anti-D (si el feto fuese Rh +) como una isoinmunización anti-A
Recuerda otras peculiaridades: (si el feto fuese A). Obviamente, nos preocupará más el posible
• Las manifestaciones neurológicas son más frecuentes en las desarrollo de una isoinmunización anti-D. Esta isoinmunización
sepsis tardías. requiere una sensibilización previa (la isoinmunización anti-ABO
• Los agentes etiológicos más frecuentes son: SGB y E. coli. no la requiere), esto es, un primer embarazo o un primer aborto
• La mortalidad es mayor en las sepsis precoces que en las tar- con un fruto de concepción Rh +. Para despistar este hecho, se
días. recurre al test de Coombs indirecto, que detecta anticuerpos anti-D
• El tratamiento empírico ha de incluir, al menos 7 días. en sangre materna:

CTO Medicina • C/Núñez de Balboa 115 • 28006 Madrid • Tfno.: (0034) 91 782 43 30/33/34 • E-mail: secretaria@ctomedicina.com • www.grupocto.es
5
Pediatría
Test 2V Distancia
• Si el test de Coombs indirecto es negativo, implica que la madre no
está sensibilizada. Podemos hacer profilaxis. A través de la profilaxis,
Comentarios
Grupo CTO
CTO Medicina

Las necesidades de líquidos dependen de la edad gestacional, las


condiciones ambientales, y la existencia de enfermedades. Las pér-
se pretende hemolizar terapéuticamente los hematíes fetales que didas insensibles son mayores cuanto menor sea la edad gestacional.
hayan cruzado la placenta, para evitar con ello la sensibilización En los nacidos a término los aportes de líquidos el primer día son de
de la madre (para evitar que la madre fabrique anticuerpos anti-D 60-70 ml/kg/día, y se aumenta a 100-120 ml/kg/día en los días 2 y 3,
por sí misma). Opción 1, correcta. mientras que en los prematuros se empieza con 70-100 ml/kg/día y
• Si el test de Coombs indirecto es positivo, implica una sensibiliza- se llega a los 150 ml/kg/día a los 3-4 días.
ción previa, con lo cual la prevención con Ig anti-D resulta inútil.
Opción 4, falsa. Pregunta 34.- R: 3
En el ombligo del recién nacido debemos distinguir la presencia
Ante cuadros muy severos de isoinmunización anti-D, puede llegar de alteraciones típicas:
a ser necesario la transfusión de hematíes al feto (opción 2, correcta). • La persistencia del uraco y del conducto onfalomesentérico pro-
Para valorar el grado de hemólisis intraútero, se recurre al análisis ducen secreción líquida por el ombligo, ácida en el primer caso,
espectrofotométrico del líquido amniótico. La zona III implica un y alcalina en el segundo.
alto riesgo fetal (opción 5, correcta). Las zonas I y II implican menor • El granuloma (el caso clínico corresponde a ello), y el pólipo umbi-
riesgo. lical aparecen como un tejido exuberante, un tumoración a nivel
del ombligo, no dolorosa, en el primer caso rosada, blanda y con
Pregunta 31.- R: 3 secreción serosa, y en el último, roja oscura, dura y con secreción
La policitemia neonatal se asocia con diversas patologías: trans- mucoide. El diagnóstico es clínico, realizándose biopsia si hay mu-
fusión materno-fetal o feto-fetal, retraso en la ligadura del cordón, chas dudas, siendo el tratamiento del granuloma la cauterización,
CIR, recién nacidos postérmino, hijo de madre diabética, hiperplasia y el del pólipo la extirpación completa.
adrenal, síndrome de Down, trisomías 13 y 18, hipotiroidismo. • La hernia umbilical se produce por un cierre imperfecto o debili-
Se diagnostica a través del hematocrito central (mayor del 65%); el dad del anillo umbilical, produciendo una protrusión de la pared
hematocrito periférico suele ser mayor que el central, y no se considera abdominal que contiene epiplon, fácilmente reductible y que la
un indicador fiable, sino aproximativo. mayoría de las veces se resuelve espontáneamente. Si no lo hace,
El problema de la policitemia es que provoca hiperviscosidad, o la hernia es muy importante, se cierra quirúrgicamente.
y enlentece y dificulta la circulación hemática. Este evento puede • Las infecciones umbilicales en el período neonatal tiene riesgo de
derivar en patología neurológica (hiperexcitabilidad) e insuficiencia diseminación hematógena, al hígado o al peritoneo, y hay que tratar-
cardíaca. las siempre agresivamente, con antibioterapia IV. Se manifiestan por
eritema periumbilical, con secreción purulenta maloliente en algunos
Pregunta 32.- R: 5 casos, y signos de infección sistémica en casos avanzados.
Un RNPT de bajo peso al nacer, presenta un cuadro compatible
con una crisis motora focal. En la TC craneal aparecen ventrículos Pregunta 35.- R: 3
ligeramente dilatados y calcificaciones periventriculares. ¿Qué Señale la afirmación FALSA, de entre las siguientes, respecto a
patología es más probable que presente este RN? la ictericia fisiológica:
1) Sífilis congénita. 1) Las cifras máximas de bilirrubina suelen alcanzarse a los 3-4 días.
2) Toxoplasmosis congénita. 2) La ictericia en los RNPT suele ser de inicio algo más tardío y
3) Rubéola congénita. más prolongada que en los RNT.
4) Varicela congénita. 3) Puede comenzar en las primeras 24 horas de vida en el RNT.
5) Infección prenatal por citomegalovirus. 4) Su duración suele ser inferior a 10-15 días.
5) En los RNPT pueden alcanzarse cifras de hasta 14 mg/dl.
El citomegalovirus es la causa más frecuente de infección congé-
nita. Si la infección se adquiere en la primera mitad del embarazo, Para que una ictericia se considere fisiológica debe aparecer una
aparece un cuadro de coriorretinitis, calcificaciones periventriculares vez pasadas las primeras 24 horas de vida (respuesta 3 incorrecta). La
(respuesta 5 correcta) y microcefalia. Es más frecuente que la infección duración de la misma es inferior a 10-15 días. Las cifras de bilirrubina
se contraiga en la segunda mitad de la gestación. En este caso, suele total no suelen superar los 12 mg/dl en RNT o los 14 mg/dl en RNPT.
cursar de forma asintomática, pero puede derivar en una hipoacusia En los RNPT la ictericia fisiológica suele ser de inicio algo más tardío
neurosensorial bilateral. que en el RNT y también suele ser algo más prolongada.

Pregunta 33.- R: 4 Pregunta 36.- R: 4


La supervivencia de los nacidos de muy bajo peso (prematuros la RN a término al que se le determina el valor de TSH en sangre,
inmensa mayoría), aumenta en un ambiente térmico neutro, que es obtenida mediante la prueba del talón, el día 3 de vida. El valor de
el que necesitan para mantener una temperatura corporal de 36.5- la TSH aparece elevado. ¿Qué actitud considera más adecuada?:
37ºC. La temperatura de la incubadora debe regularse para ello y no 1) Repetir la prueba dentro de un mes.
ser un valor fijo; cuanto más pequeño y más inmaduro sea el niño, 2) Sospechar hipertiroidismo e iniciar tratamiento con antitiroideos.
más temperatura necesitará. La humedad de la incubadora debe 3) Tranquilizar a la madre porque se trata de algo habitual en los RN.
estar entre un 40 y un 60%, pues se ha visto que en estas condicio- 4) Sospechar hipotiroidismo congénito e iniciar tratamiento susti-
nes disminuye la pérdida corporal de calor y se consigue una mejor tutivo con hormona tiroidea.
estabilidad térmica, así como una menor irritación mucosa en el caso 5) Ingresar al RN en observación.
de administración de oxígeno suplementario.
El hipotiroidismo congénito se puede diagnosticar por la clínica. Sin
La alimentación del prematuro debe iniciarse de una forma cau- embargo, hoy en día se realiza el cribado neonatal determinando los
telosa y gradual, y si el niño no tiene dificultad respiratoria ni otros valores de TSH en sangre obtenida entre los 2 y los 5 días de vida. Si los
signos patológico, y tiene movimientos de succión, puede intentarse valores de TSH están elevados hay que sospechar hipotiroidismo congénito
la alimentación vía oral. e iniciar cuanto antes tratamiento sustitutivo con hormona tiroidea.
Aunque los prematuros tienen riesgo aumentado de lesión hipóxica
y de insuficiencia respiratoria, la administración de oxígeno conlleva Pregunta 37.- R: 3
riesgo de toxicidad pulmonar y ocular, por lo que la oxigenoterapia Recuerda otro de los “retratos robot” de las múltiples enfermedades
se administrará a los prematuros que la necesiten, teniendo en cuenta que te pueden preguntar en el MIR:
estos efectos adversos, y no de forma rutinaria en todos ellos. • Decaimiento, hipoactividad.
• Cara empastada, poco expresiva.
En efecto la leche materna, siempre que no haya contraindica- • Macroglosia.
ciones específicas, es la preferida para la alimentación del lactante, • Ictericia.
aunque sea prematuro (opción 4, correcta). En este caso, se añaden • Hernia umbilical.
suplementos de proteínas, calcio y fósforo, y si no se dispone de leche • Fontanelas aumentadas de tamaño (el neonato del enunciado
materna, se administran fórmulas especiales de prematuros, que no tiene una fontanela anterior normal, pero la fontanela posterior
deben utilizarse por encima de la semana 34-36 de edad gestacional, es amplia).
pues pueden producir hipercalcemia. Este es el cuadro típico del hipotiroidismo congénito.

CTO Medicina • C/Núñez de Balboa 115 • 28006 Madrid • Tfno.: (0034) 91 782 43 30/33/34 • E-mail: secretaria@ctomedicina.com • www.grupocto.es
6
roidismo.
Comentarios
Grupo CTO
CTO Medicina

• El hipertiroidismo no comparte ningún síntoma con el hipoti-


Test 2V
Distancia

TALLA BAJA FAMILIAR


Pediatría
RETRASO CONS-
TITUCIONAL DEL
• La fenilcetonuria cursa con decaimiento, pero no con ictericia, CRECIMIENTO
macroglosia ni hernia.
• El raquitismo comparte con el hipotiroidismo tan sólo las fon- Talla RN Menor de la normal Normal
tanelas aumentadas de tamaño.
• La hiperplasia suprarrenal tampoco comparte ningún síntoma Velocidad de Menor de lo normal,
Normal
con el hipotiroidismo. crecimiento luego normal
Antecedentes De pubertad retra-
Pregunta 38.- R: 5 De talla baja
familiares sada
• La leche de madre y la leche de vaca tienen la misma cantidad
de grasa pero ésta es cualitativamente distinta. La leche de Igual a edad talla
Igual a edad cronoló-
madre presenta más colesterol, ácidos grasos esenciales y ácidos Edad ósea
gica
Retrasada respecto a
grasos poliinsaturados de cadena larga. la edad cronológica
• El flúor es más abundante en la leche materna que en la de Normal o algo dismi-
vaca. Talla final Disminuida
nuida
• La leche de madre presenta una relación caseína/seroproteínas
de 30/70. La de vaca, de 80/20. Pregunta 39. Talla baja familiar vs retraso
• La leche de vaca presenta más cantidad de vitamina K que la constitucional del crecimiento
leche de madre (por ello, los alimentados con fórmula adaptada
presentan menor riesgo de desarrollar enfermedad hemorrágica Pregunta 40.- R: 2
del RN). Este caso clínico presenta:
• Un lactante.
• Con diarrea florida de características no cólicas (piensa en la
posibilidad etiológica del Rotavirus),
Hipotómica Isotómica Hipertómica • Signos clínicos de deshidratación intracelular (piel y mucosas
secas, hiperexcitabilidad neurológica),
Osmolaridad < 280 280-310 > 310 • Acidosis metabólica (pH<7,35 con Bicarbonato<21 mEq/l),
• E hipernatremia (Na>145 mEq/l).
Frecuencia 5-10 % 65 % > 310 Toda deshidratación hipernatrémica debe corregirse en 48-72
horas, pues una corrección rápida de la misma podría derivar en la
Afectación
Extracelular Extracelular Inracelular aparición de una mielinólisis central pontina.
fundamental
Hipotensión, Enfermedad de
hipovolemia, Funcional (adquirido)
Sed, fiebre, Hirschprung
fontanella
Similar a loguria, Historia
Clínica hundida, signo
hipotónica irritabilidad,
del pliegue, Comienzo del Pasados los 2 años de Desde el nacimiento
convulsiones
ojos hundidos, estreñimiento edad
shock Incontinencia Frecuente Muy rara
fecal (encopresis)
Riesgo de shock ↑ Normal Casos graves Esfuerzo en la Habitual Ninguno
defecación
Natremia < 130 130-150 > 150 Diámetro de las Muy grande Pequeño
heces Nunca Posible
Sodio total ↓ ↓ ↓ Enterocolitis Frecuente Frecuente
Dolor abdominal
Calcemia ↓ Normal / ↑ ↑ Exploración
Distensión
Pregunta 38. Diferencias proteicas entre la leche humana y de vaca abdominal Rara Frecuente
Desarrollo escaso Raro Frecuente
Tono anal Permeable Tenso, difícil de relajar
Pregunta 39.- R: 5 Tacto rectal Ampolla con heces Ampolla vacía
Acude a su consulta una madre con su hija de 10 años preocu- Desnutrición No Posible
pada porque dice que su hija “siempre ha sido la más bajita de
Pruebas compl.
toda la clase”. Su talla actual se encuentra en el percentil 2 de la
población general. Su talla genética está situada en el percentil
Enema opaco Masiva cantidad de Zona de transición,
3. Su desarrollo sexual es el que corresponde para su edad y su
heces, no hay zona de demora de la evacua-
edad ósea es de 9 años. ¿Qué situación cree usted que presenta
transición ción (más de 24 horas)
la niña?:
1) Un hipocrecimiento por deficiencia de la hormona de creci- Ausencia de células
miento. Biopsia rectal Normal ganglionares
2) Un hipocrecimiento por un síndrome de Turner. Nula relajación del
3) Un hipocrecimiento por un hipotiroidismo congénito. Manometría La distensión del recto esfínter
4) Un retraso constitucional del crecimiento y del desarrollo. anorrectal produce relajación del
5) Una talla baja familiar. esfínter interno
Pregunta 40. Tipos de deshidratación
La talla baja familiar es una de las causas más frecuentes de talla
baja. La curva de crecimiento es igual o inferior al percentil 3, dis- Pregunta 41.- R: 1
curriendo de forma paralela, además existe historia familiar de talla En el raquitismo las primeras alteraciones se dan en el plano ra-
baja. La pubertad se produce a la edad habitual y no hay discordancia diológico: pobre mineralización ósea, metáfisis en copa, incurvación
entre la edad ósea y la edad cronológica. de los huesos largos, zonas de Looser-Milkmann.
La talla adulta final es baja, pero dentro de los límites de lo es- Las necesidades diarias de vitamina D para un RN normal son 400
perado para su talla genética. Todas las pruebas de laboratorio son UI. Aunque estamos en un país suficientemente soleado, prácticamen-
normales. te todos los pediatras pautan para sus lactantes vitamina D desde los

CTO Medicina • C/Núñez de Balboa 115 • 28006 Madrid • Tfno.: (0034) 91 782 43 30/33/34 • E-mail: secretaria@ctomedicina.com • www.grupocto.es
7
Pediatría
Test 2V Distancia
primeros 15-30 días hasta el año de vida, si bien a título teórico esta
actitud no sería necesaria.
Comentarios
Grupo CTO
CTO Medicina

• La exploración de un lactante con bronquiolitis muestra durante


la inspección signos de dificultad respiratoria (tiraje supraes-
Los niños de raza negra padecen más raquitismo carencial que los ternal, subcostal e intercostal), y a la auscultación, polipnea,
de raza blanca, por motivos esencialmente socioeconómicos. espiración alargada, sibilancias, crepitantes espiratorios e hi-
La opción 1 es falsa. La craneotabes parietal es fisiológica en los poventilación.
RN, no así la occipital, considerada patológica siempre. • Es raro que un lactante con bronquiolitis se sobreinfecte por
bacterias.
Pregunta 42.- R: 2 • Un 20% de los lactantes que padecieron bronquiolitis presentará en
La opción 1 es correcta. En el retraso constitucional del crecimiento un futuro hiperreactividad de las vías aéreas durante la infancia.
y la pubertad, al igual que en la talla baja familiar, los niveles de GH
son normales. La opción 2 es falsa. En la TBF la edad ósea coincide Pregunta 47.- R: 5
con la cronológica. La opción 3 es correcta. En el RCCP la edad ósea Este caso clínico te puede poner en la encrucijada de dos diagnós-
es menor que la cronológica. La opción 4 es correcta. En el RCCP la ticos diferenciales: epiglotitis aguda y absceso retrofaríngeo.
talla final es normal o levemente disminuida. La opción 5 es correcta. Podría ser una epiglotitis, por la conjunción de dificultad respirato-
En la TBF la pubertad no aparece retrasada. ria, fiebre alta y disfagia. Pero en las epiglotitis no se ve ninguna masa
en la pared posterior de la faringe.
Pregunta 43.- R: 2 En el absceso retrofaríngeo (opción correcta), que casi siempre
Recuerda que, para el MIR, has de tener muy en cuenta las dife- supone una complicación de una faringoamigdalitis, sí se ve una masa
rencias entre: en la pared posterior de la faringe. A nivel clínico destaca que el niño
• Laringotraqueítis vírica: disfonía, estridor, tos perruna, dificultad se niega a comer (por intenso dolor), tiene fiebre alta, mal aspecto, y
respiratoria leve-moderada, empeoramiento nocturno y fiebre. muestra rigidez cervical.
• Laringitis espasmódica, igual que la anterior pero sin fiebre.
• Traqueítis bacteriana: laringitis que progresa torpemente, con Descartadas quedarían, de entrada el resto de las opciones:
empeoramiento del estado general, fiebre alta y mayor dificultad • Las faringoamigdalitis aisladas no provocan distrés ni asocian
respiratoria. masa retrofaríngea.
• Epiglotitis: Comienzo súbito, rápidamente progresivo, con mal • La sofocación por cuerpo extraño no provoca fiebre.
estado general, importante dificultad respiratoria, dislalia en patata • En la traqueítis bacteriana no hay disfagia ni masa faríngea.
caliente, disfagia e hiperextensión cervical.
Pregunta 48.- R: 3
Pregunta 44.- R: 4 La dificultad respiratoria infantil se puede clasificar conceptual-
Esta pregunta es para no fallarla: hay que identificar un caso de mente en dos tipos:
laringotraqueítis vírica complicado con una traqueítis bacteriana. Las • Inspiratoria. Bajo ella subyace patología laríngea. Su elemento
complicaciones de la laringitis vírica son excepcionales, hay que recor- auscultatorio clave es el estridor inspiratorio.
dar ésta, que es la más característica. La traqueítis bacteriana es más • Espiratoria. Bajo ella subyace patología del árbol bronquial. Su
frecuente en niños menores de 3 años, está producida en la mayoría fenómeno auscultatorio clave es la sibilancia. El primer episodio
de las ocasiones por Staphylococcus aureus, generalmente aparece tras de dificultad respiratoria sibilante en pacientes de menos de dos
una infección respiratoria vírica (casi siempre laringitis aguda), produce años se define como bronquiolitis; la recurrencia de episodios
afectación del estado general, fiebre alta y dificultad respiratoria. El sibilantes se conoce como asma (obstrucción reversible de la
diagnóstico es clínico, sin que haya alteraciones analíticas ni radioló- vía aérea asociada a inflamación y broncoespasmo). El asma
gicas específicas. El tratamiento de elección son los antimicrobianos infantil puede obedecer a procesos infecciosos o a fenómenos
antiestafilocócicos (cloxacilina, amoxicilina-clavulánico; vancomicina de alergia.
si se sospecha una cepa meticilín resistente). La intubación puede
ser necesaria en los casos graves, pero no se realiza de rutina. El En nuestro paciente, sin desencadenante alguno (así queda descar-
pronóstico es bueno, mejorando la mayoría notablemente tras 2-3 tada la aspiración de cuerpo extraño al árbol bronquial, cuadro que
días de tratamiento antibiótico. Como otras infecciones que produce entraría en el diagnóstico diferencial de la tos súbita con sibilancias,
el Staphylococcus aureus, puede ir asociada a un síndrome de shock y que tiene una primera fase de sofocación), aparece tos, dificultad
tóxico, patología producida por la toxina de esta bacteria. respiratoria, sibilancias y signos radiológicos de atrapamiento aéreo.

Pregunta 45.- R: 1 La opción 1 es falsa, pues el paciente tiene más de dos años. La
La bronquiolitis es una enfermedad frecuente de las vías respira- opción 2 es falsa, pues la tos psicógena cursa sin sibilancias y sin
torias, que afecta a niños menores de 2 años de edad, especialmente radiología de atrapamiento. La opción 4 es falsa, pues el paciente
a lactantes menores de 6 meses, que se presenta de forma estacional, está afebril, y no tiene infiltrados intersticiales en la placa de tórax. La
con un pico en invierno, y que plantea el diagnóstico diferencial opción 5 es falsa, pues el crup suele afectar a niños más pequeños,
principalmente con la hiperreactividad bronquial. Los antecedentes presenta estridor inspiratorio y no cursa con sibilancias.
familiares de asma, el comienzo brusco de los síntomas no precedido
de infección, la eosinofilia, la repetición de episodios similares, apoyan Pregunta 49.- R: 2
el diagnóstico de broncoespasmo. En la radiografía de tórax en ambos Kevin es un niño de cuatro años, que acude a urgencias del
casos podemos encontrar hiperinsuflación pulmonar, siendo frecuente hospital en el que Vd. está de guardia a las dos de la madrugada.
además en los casos de bronquiolitis la presencia de infiltrados pul- La madre refiere que desde la tarde se queja de dolor de garganta
monares y áreas de consolidación por atelectasias. y presenta fiebre de 39,5ºC. Cuando Vd. lo ve, está sentado, con
Mientras que el tratamiento del broncoespasmo son los bronco- la boca abierta, presentando salivación intensa. Con respecto al
dilatadores (beta 2 agonistas, anticolinérgicos), y los corticoides en los proceso que Vd. sospecha, señale la opción FALSA, de entre las
casos moderados-graves, ninguno de ellos ha demostrado beneficio siguientes:
en la bronquiolitis. Sólo la adrenalina nebulizada, ha demostrado 1) Es probable que Kevin no haya completado el calendario vacunal.
una mejoría muy discreta y momentánea, sin influir en la evolución 2) Este cuadro clínico no suele precisar intubación endotraqueal.
de la enfermedad, por lo que tampoco está indicado su uso clara- 3) En la Rx lateral de faringe se podrá apreciar la epiglotis aumen-
mente. Lo más importante en el tratamiento de la bronquiolitis son tada de tamaño.
las medidas físicas y la oxigenoterapia cuando la saturación del niño 4) El tratamiento farmacológico de elección es la cefotaxima iv.
no es la adecuada. 5) Pueden aparecer como complicación otras infecciones, como
la otitis media o la neumonía.
Pregunta 46.- R: 4
Recuerda algunos datos relacionados con la bronquiolitis: Caso clínico típico de epiglotitis aguda que se debe sospechar
• El adenovirus provoca una bronquiolitis especial, conocida por la presencia de fiebre alta, dolor de garganta, babeo y dificultad
como síndrome del pulmón hiperclaro unilateral (esto lo han respiratoria en un niño que prefiere estar sentado e inclinado hacia
preguntado en el MIR). delante. Actualmente los principales patógenos responsables del
• El VRS provoca infecciones respiratorias inocentes en adultos y cuadro son cocos gram positivos (S. pyogenes, S. pneumoniae y S.
niños mayores, muchos de los cuales actúan como “vector” de aureus), por detrás de éstos se encuentra el agente clásico, el H.
la infección sobre los lactantes. influenzae tipo B.

CTO Medicina • C/Núñez de Balboa 115 • 28006 Madrid • Tfno.: (0034) 91 782 43 30/33/34 • E-mail: secretaria@ctomedicina.com • www.grupocto.es
8
Comentarios
Grupo CTO
CTO Medicina

Si fuera necesario se realizaría una Rx lateral de laringe donde


se podría visualizar una epiglotis hinchada. Es un cuadro muy grave
Test 2V Distancia Pediatría
• Si se deja evolucionar una invaginación, con el tiempo aparecerá
compromiso de irrigación, necrosis y deposiciones hemorrágicas.
donde suele ser necesario establecer una vía aérea artificial a través • Se suele diagnosticar a través de ecografía abdominal.
de la intubación nasotraqueal (respuesta 2 falsa) junto con cefotaxima • Sólo un 5-8% de los casos tratados con enema recidivan (no
endovenosa. En el curso de la epiglotitis pueden aparecer complica- un 45%).
ciones como neumonía, linfadenitis cervical y otitits. • El signo del muelle enrollado en el enema opaco apoya el
diagnóstico.
Pregunta 50.- R: 2
La fibrosis quística es una enfermedad multisistémica caracte- Pregunta 54.- R: 2
rizada fundamentalmente por obstrucción e infección de las vías El diagnóstico es el de atresia esofágica con fístula traqueoesofá-
respiratorias y por mala digestión. Se hereda de forma autonómica gica distal, el tipo más frecuente (más del 85%) de todos. En los casos
recesiva, y es más frecuente en la raza caucásica. Las mutaciones que en que hay fístula proximal, el abdomen está excavado, porque no
cusan la enfermedad son diversas, pero todas afectan al mismo locus llega al tubo digestivo aire ni por el esófago, ni por la vía respiratoria,
del brazo largo del cromosoma 7; en Europa, el 50% de los pacientes y pueden deglutir, pero cada vez que lo hacen tienen un episodio
aproximadamente, son homocigotos para la delección F508. de aspiración grave. La mayoría de las veces el diagnóstico se hace
La fibrosis quística es la causa más frecuente de neumopatía en el paritorio, porque no se logra pasar la sonda al estómago, una
crónica en la infancia y de insuficiencia pancreática exocrina, y ade- práctica de rutina, y generalmente la radiografía de tórax es suficiente
más produce otras alteraciones como síndrome pierde sal, poliposis para confirmarlo, viéndose la sonda enrollada en la bolsa esofágica
nasal, pansinusitis, prolapso rectal, pancreatitis, colelitiasis, diabetes superior.
mellitus.
El objetivo principal del tratamiento es mantener al paciente estable, Hay que sospechar la atresia esofágica en los siguientes casos: poli-
sin reagudizaciones de la enfermedad, el mayor tiempo posible, siendo la hidramnios materno; la sonda de reanimación en el paritorio no pasa
terapia pulmonar y nutricional lo más importante. La primera se fundamenta al estómago; exceso de secreciones orales desde el nacimiento; tos,
en la antibioterapia, oral y nebulizada, siendo muy importante la identifi- cianosis o atragantamiento con los intentos de alimentación. El 30%
cación de los microorganismos que producen las exacerbaciones, y ajustar de los afectados nacen prematuros y el 50% tienen malformaciones
en lo posible el espectro antibacteriano al resultado de los antibiogramas. asociadas (recordad el acrónimo VACTERL: vertebrales, anorrectales,
Cuando el paciente desarrolla síntomas progresivos o no mejora a pesar cardíacas, traqueales, esofágicas, renales, y del radio).
de la antibioterapia oral y nebulizada, está indicada la intravenosa, siendo
frecuente su administración domiciliaria, en caso de que el paciente no El tratamiento es quirúrgico, urgente, necesitándose en ocasiones un
precise otros cuidados hospitalarios. segundo tiempo para la anastomosis esofágica (lo primero, es cerrar la
fístula). Las secuelas son frecuentes: estenosis esofágica, atragantamientos,
Pregunta 51.- R: 4 lentitud para alimentarse, y como consecuencia fallo de medro.
El primogénito de una pareja sufre en el período neonatal íleo
meconial. ¿Cuál es el método de diagnóstico principal que llevaría Pregunta 55.- R: 3
a cabo para establecer el diagnóstico en la enfermedad subyacente Este caso clínico presenta:
más probable?: • Un niño de menos de dos años.
1) Test del tripsinógeno inmunorreactivo. • Con retraso ponderal,
2) Test de meconio. • Deposiciones de características esteatorreicas,
3) Estudio molecular genético directo. • Sintomático en forma de anorexia, postración y mal talante,
4) Test de cloro en sudor. • Anemia ferropénica (ferritina y sideremia bajas),
5) Estudio genético de vínculo o cercanía. • Y test de Van de Kammer con aumento de la grasa total.

En un neonato que presenta íleo meconial debemos sospechar Este es el “retrato robot” de la enfermedad celíaca. La actitud
fibrosis quística. El estudio molecular genético directo sería el método diagnóstica inicial consiste en la conjunción de dos exploraciones
diagnóstico principal en este caso (la existencia de dos mutaciones complementarias:
conocidas, en el brazo largo del cromosoma 7, confirmarían el diag-
nóstico). El test del tripsinógeno inmunorreactivo sérico (que estará • Determinación de anticuerpos de celíaca.
elevado si hay afectación pancreática) serviría para el cribado neonatal, • Realización de biopsia intestinal. Si ésta fuera patológica, ten-
pero no para establecer el diagnóstico definitivo. La prueba del sudor dría que retirar el gluten de la dieta. Después, como se trata
serviría para el diagnóstico, pero no en neonatos pues en las primeras de un paciente de menos de dos años, tendría que hacer dos
semanas de vida es poco fiable, por la dificultad de obtener sudor. biopsias más: la de normalización (tras la retirada del gluten) y
la de provocación (reintroduciendo el gluten).
Pregunta 52.- R: 5
Este caso clínico presenta: Pregunta 56.- R: 4
• A un lactante. La biopsia de un paciente celíaco muestra:
• Con llanto paroxístico, • Atrofia subtotal o total de las vellosidades.
• Signos localizadores abdominales (encogimiento de piernas • Hiperplasia de las criptas.
hacia el vientre), • Infiltrado linfocitario en la lámina propia.
• Signos indirectos de sufrimiento (palidez),
• Y masa en hemiabdomen derecho. Pregunta 57.- R: 4
Los anticuerpos más sensibles y específicos son los antitransglutami-
Este es el “retrato robot” de la invaginación intestinal: nasa. La positividad de los anticuerpos se correlaciona con la actividad
• La invaginación intestinal es la causa más frecuente de obstruc- de la enfermedad, y se utiliza como monitor del curso de la enfermedad
ción intestinal entre los 3 meses y los 6 años de vida. y como sensor de transgresiones dietéticas.
• Sólo en contadas ocasiones se logra advertir una causa orgánica
que lo favorezca (adenitis mesentérica, linfoma,...). Pregunta 58.- R: 3
• La localización más frecuente de la invaginación es la ileocólica. En Este caso clínico presenta:
la radiología se verá un silencio aéreo en hemiabdomen derecho. • A un lactante varón.
• Se ha de diagnosticar y tratar preferentemente en las primeras • Con síndrome de Down,
48 horas. • Síntomas respiratorios (tos nocturna),
• La localización más frecuente de la invaginación es la íleocólica, • Y síndrome de Sandifer (hipertonía cervical tras las tomas).
seguida de la íleoíleocólica. Ante este cuadro hay que pensar en una enfermedad por RGE.

Pregunta 53.- R: 4 Pasemos a analizar las distintas respuestas:


En relación con la invaginación intestinal recuerda: • La esofagitis por RGE provoca microsangrados que derivan en
• Hay que descartar que se organice sobre una lesión orgánica la aparición de anemia ferropénica.
previa, como el linfoma, sobre todo si el paciente tiene más de • El RGE es muy frecuente (1/300 RN lo presentan), sobre todo
seis años. en varones.

CTO Medicina • C/Núñez de Balboa 115 • 28006 Madrid • Tfno.: (0034) 91 782 43 30/33/34 • E-mail: secretaria@ctomedicina.com • www.grupocto.es
9
Pediatría
Test 2V Distancia
• La opción 3 es INCORRECTA. La técnica de elección en el
diagnóstico del RGE es la pHmetría, que cuantifica y define el
Comentarios
Grupo CTO
CTO Medicina

4) En la intolerancia secundaria a hidratos de carbono, la exclusión


de disacáridos es permanente.
carácter de los episodios de reflujo. 5) Una nutrición precoz con dieta normal en pacientes que han
• En los casos leves, basta con establecer medidas dietético- sufrido una GEA evita su aparición.
posturales.
• Los síntomas respiratorios incluyen: tos crónica, laringitis de re- En la diarrea por malabsorción de azúcares las deposiciones son
petición, bronquitis de repetición y neumonía por aspiración. ácidas (pH menor de 5,5), asocian dolor abdominal, aumento de
los ruidos hidroaéreos y provocan eritema perianal. El Clinitest, que
Pregunta 59.- R: 4 detecta la presencia de azúcares reductores en heces, es positivo en
Has de tener en cuenta que estreñimiento que comienza desde el la malabsorción de lactosa. El síndrome postenteritis (déficit transito-
nacimiento+retraso ponderoestatural+antecedente de retraso en la rio de lactasa secundaria a un proceso de gastroenteritis aguda) es la
evacuación de meconio es igual a enfermedad de Hirschprung. causa más frecuente de la malabsorción de la lactosa. Este síndrome
• La alteración más típica del enema opaco de los pacientes con se puede intentar evitar si se hidrata y se nutre precozmente con
enfermedad de Hischprung es la presencia de un recto cuyo una dieta normal a los pacientes con GEA. El tratamiento consiste
calibre es la mitad del segmento previo a la estenosis. en la retirada de la lactosa de la dieta hasta la remisión del cuadro
• En la manometría, el evento más característico es la presencia (respuesta 4 incorrecta).
de una contracción del esfínter anal interno ante aumento de
las presiones en territorios proximales a él. Pregunta 63.- R: 4
• El diagnóstico definitivo lo da la biopsia rectal (ausencia de Recuerda que un varón que presenta desde la tercera semana
células ganglionares con aumento de las terminaciones nerviosas de vida vómitos alimentarios (jamás biliosos), proyectivos, tras todas
y de la actividad de la acetilcolinesterasa). las tomas, es igual a estenosis hipertrófica de píloro. Todo cuadra
• El tratamiento de elección es la cirugía. especialmente si en el enunciado te comentan que presenta una
masa con forma de aceituna en epigastrio/hipocondrio derecho, o si
te aportan datos analíticos de alcalosis hipoclorémica.
GRADO DEFINICIÓN TRATAMIENTO Pregunta 64.- R: 4
Se estima que hasta un 10% de los preescolares y los escolares
Reflujo a uréter no padecen “dolor abdominal recurrente”. En los menores de 2 años,
I dilatado. Resolución
espontánea del reflujo. casi siempre se debe a una causa orgánica, pero en los niños mayores,
Reflujo a sistema colector Profilaxis ITU. sólo en el 10% se encuentra una causa orgánica. Aunque siempre
II no dilatado. hay que pensar en la posibilidad de que haya una patología orgánica
Dilatación ureteral Vigilar + plantearse
subyacente, en los casos en los que la anamnesis y la exploración física
III (cálices redondeados). cirugía. sugieren un origen funcional (no orgánico) del dolor, no es necesario
realizar pruebas complementarias. En estos casos es muy importante
Dilatación intentar tranquilizar a la familia y al niño para su tratamiento, siendo
IV ureteral grave. de peor pronóstico que los de causa orgánica.
Abombamiento pieloca- Cirugía. Aunque la mejoría durante los fines de semana y las vacaciones van
V licial severo, dilatación y a favor del dolor abdominal funcional, este patrón puede estar ausente,
tortuosidad de uréteres. y el niño incluso puede despertarse por la noche quejándose del dolor.
Habitualmente el dolor abdominal funcional se localiza en la región
Pregunta 59. Características diferenciales de la enfermedad de periumbilical y es difícil que el niño lo defina claramente. Salvo que los
Hirschprung y del estreñimiento funcional (adquirido) síntomas sugieran una enfermedad péptica, no está indicado realizar
pruebas de detección de H. pylori, pues la prevalencia de infección
crónica por esta bacteria en los niños con dolor abdominal recurrente
Pregunta 60.- R: 2 es la misma que en la población general.
La diarrea crónica inespecífica es una modalidad inocente de
diarrea crónica que ocurre en algunos niños entre los 6 meses y los Pregunta 65.- R: 5
3-4 años de vida, y que es un equivalente del colon irritable del La manifestación más frecuente del divertículo de Meckel es
adulto, con el cual comparte patogenia (ambas son diarreas motoras) la hemorragia indolora. También puede provocar dolor abdominal
y familiaridad (los hijos de padres con colon irritable tienen mayor agudo y fenómenos de diverticulitis. Todos estos fenómenos se deben
probabilidad de desarrollar diarrea crónica inespecífica). a la presencia de mucosa gástrica ectópica, que genera un ambiente
ácido que erosiona la vecindad del divertículo. Para el diagnóstico del
Recuerda para el MIR: diarrea de curso crónico (con deposiciones divertículo de Meckel es de gran valía la gammagrafía con Tc99 (que
que contienen fibras vegetales sin digerir y moco), que no asocia detecta únicamente aquellos divertículos en los cuales hay ectopia
retraso ponderal es igual a diarrea crónica inespecífica. gástrica). Para aumentar la validez de esta prueba se puede utilizar
La actitud a seguir consiste en serenar a los padres, y explicarles el fármacos u hormonas que son dianas para las células gástricas (cime-
carácter autolimitado de este tipo de diarrea. tidina, glucagón, gastrina).
Pregunta 61.- R: 5 Pregunta 66.- R: 4
Recuerda para el día del MIR que: lactante + manifestaciones ¿Cuál de las siguientes características es más frecuente y peculiar
alérgicas (habones, angioedema, broncoespasmo) + síntomas diges- de los vómitos por reflujo gastroesofágico (calasia) en el niño?
tivos (vómitos, diarrea, rectorragia) es igual a alergia a proteínas de 1) Asociación con ferropenia.
leche de vaca. La actitud terapéutica a seguir con estos pacientes es 2) Relación con la ingesta.
retirar la fórmula adaptada que estaba tomando, y sustituirla por un 3) Repercusión sobre el crecimiento.
hidrolizado de proteínas. Si el paciente tomaba leche de madre, basta 4) Comienzo neonatal y mejoría con la edad.
con retirar de la dieta de la mamá los lácteos. 5) Asociación con síntomas de broncoconstricción.
Pregunta 62.- R: 4 El reflujo gastroesofágico fisiológico es la modalidad de reflujo que
Niño de 13 meses, que tras padecer una gastroenteritis aguda afecta a niños menores de 2 años, cursa con síntomas típicos (como
hace una semana, presenta una diarrea líquida que produce erite- el vómito atónico o regurgitación tras las tomas) y no produce retraso
ma en la zona del pañal. Al explorar al niño advierte un aumento ponderoestatural. Este reflujo no complicado desaparece en más de la
de los ruidos hidroaéreos. Respecto a esta patología qué opción mitad de los casos a los 2 años (respuesta 4 correcta) pues se adopta
es incorrecta: la posición erecta y se modifica la consistencia de los alimentos.
1) El pH fecal suele ser menor de 7.
2) La prueba del Clinitest es positiva en la malabsorción de hidratos Es importante diferenciar el caso anterior de la enfermedad por
de carbono del tipo lactosa. reflujo gastroesofágico en la que, junto al reflujo, aparecen más
3) La causa más frecuente de deficiencia secundaria de lactasa es situaciones patológicas como: escasa ganancia ponderal, clínica
la diarrea infecciosa. respiratoria, esofagitis, etc.

CTO Medicina • C/Núñez de Balboa 115 • 28006 Madrid • Tfno.: (0034) 91 782 43 30/33/34 • E-mail: secretaria@ctomedicina.com • www.grupocto.es
10
Pregunta 67.- R: 3
Comentarios
Grupo CTO
CTO Medicina

Los datos que han de ponerte en el camino diagnóstico del sín-


Test 2V Distancia Pediatría
• El hecho patogénico básico es la ausencia del factor estimulante
de la producción de prostaciclinas.
drome de Reye son: • La biopsia renal sólo está indicada en casos de insuficiencia renal
• Niño en edad escolar. prolongada o cuando ésta no coexiste con trombopenia.
• Antecedente de varicela/gripe. • La función renal se recupera en un 90% de los pacientes con
• Consumo de AAS. un tratamiento adecuado.
• Datos de encefalopatía (letargia, sonmolencia, vómitos). • La recidiva del SHU es infrecuente.
• Datos de afectación hepática (hipertransaminasemia, hipera-
moniemia, hipoglucemia) sin ictericia. Pregunta 72.- R: 4
• Aumento de la glutamato deshidrogenasa. En relación con el reflujo vesicoureteral en la infancia, señale
la respuesta FALSA:
Pregunta 68.- R: 2 1) El reflujo vesicoureteral primario se debe a una anomalía con-
En relación con el síndrome de Reye debes recordar: génita de la unión ureterovesical.
• Hay metabolopatías de la ruta de la beta-oxidación y del ciclo 2) En los casos de duplicación ureteral, el reflujo es más frecuente
de la urea que cursan con clínica similar (acidosis, hipoglucemia, en el uréter que se introduce en la vejiga en el punto más alto
vómitos, letargia). Esto es así porque comparten la organela y lateralizado.
disfuncional: la mitocondria. 3) En el reflujo de grado I y II el 80% de los casos se resuelven
• La afectación neurológica de cualquier metabolopatía es difusa, espontáneamente.
nunca focal. 4) En el reflujo de grado IV y V la cirugía debe demorarse, bajo
• El síndrome de Reye aparece en algunos escolares con gripe/ profilaxis antibiótica, hasta asegurarse plenamente que el reflujo
varicela que usan como antipirético el AAS. no desaparece.
• Las medidas antiedema (manitol, hiperventilación controlada, 5) Los resultados de la cirugía son excelentes.
coma barbitúrico, restricción de líquidos) son clave, junto con
el adecuado aporte de glucosa, vitamina K y plasma fresco En el reflujo de grado I y II en pacientes que no presentan dilatación
congelado. ureteral, la anatomía de la región vesicoureteral suele ser casi normal
y en aproximadamente el 80% de los casos, el reflujo desaparece de
Pregunta 69.- R: 1 forma espontánea al madurar el niño. En estos casos la profilaxis anti-
Cuando se diagnostica de fimosis a un lactante, al comentarles biótica permanente es el pilar básico de tratamiento. En los grados IV
sobre el proceso a sus padres tenemos que tener en cuenta que: y V es de esperar que las repercusiones morfológicas no desaparezcan
1) Hasta los dos años se puede tratar de un proceso fisiológico. y/o lesionen más el riñón, por lo que se prefiere la cirugía desde un
2) Es un hallazgo poco frecuente. primer momento, sin demora (opción incorrecta 4).
3) Se realiza circuncisión solo en caso de existir motivos religiosos.
4) Las adherencias peneanas y la fimosis se trata de una misma Pregunta 73.- R: 4
patología. La causa más frecuente de HTA en la infancia es:
5) En la actualidad el uso de pomadas de corticoides evita la cir- 1) Coartación de aorta.
cuncisión por motivos médicos. 2) Hiperplasia suprarrenal congénita.
3) HTA esencial.
La fimosis en los lactantes varones es un hallazgo relativamente 4) Nefropatía por reflujo.
frecuente que se puede considerar como un proceso fisiológico, 5) Trombosis de la arteria renal.
distinto a las adherencias peneanas, hasta los dos años de edad (res-
puesta 1 correcta). El reflujo vesicoureteral (RVU), paso retrógrado de orina desde
la vejiga a la pelvis renal, predispone a la infección del tracto uri-
Pregunta 70.- R: 5 nario. La reacción inflamatoria desencadenada puede dar lugar
La obstrucción de la vía urinaria puede localizarse a cualquier nivel a la formación de cicatrices. Si éstas son extensas, se afectará la
desde el meato uretral hasta los cálices, dependiendo de ello sus efec- función renal (neuropatía por reflujo). Esta entidad es una de las
tos fisiopatológicos, así como de la edad de comienzo, la magnitud de causas más frecuentes de HTA en la infancia y subyace en el 30%
la obstrucción, y de su forma de instauración, aguda o crónica. de las ITU pediátricas.
En los niños, lo más frecuente es que la obstrucción sea congénita y
puede manifestarse en la vida fetal. La primera causa, por frecuencia, Pregunta 74.- R: 5
es la estenosis de la unión pielocalicial, con un amplio espectro de Un niño de 12 años y 5 meses se presenta en el servicio de
gravedad; sin embargo, en los varones, la causa más frecuente de Urgencias con dolor inguinal y escrotal intenso, que comenzó de
uropatía obstructiva grave (no sólo obstrucción), son las valvas de forma brusca después de un partido de fútbol. El niño está afec-
uretra posterior. tado y se observa tumefacción y dolor en hemiescroto derecho,
En cuanto a la estenosis pielocalicial, si es moderada-leve, y el no se detecta reflejo cremastérico y el teste derecho está elevado
riñón funciona bien, se recomienda una actitud expectante aunque y rotado. Señale la opción FALSA, respecto a este cuadro:
con controles periódicos; en muchos casos hay una mejoría espon- 1) Habitualmente ocurre en niños mayores de doce años.
tánea con el tiempo. En los casos graves, hay que poner profilaxis 2) La torsión del apéndice testicular es más frecuente entre los 2
antibiótica, y considerar el tratamiento quirúrgico, que consiste en y los 11 años.
una pieloplastia, y que tiene entre un 91 y 98% de éxitos. Las indi- 3) En niños mayores de 13 años, debe hacerse el diagnóstico
caciones son: hidronefrosis bilateral grave, tumoración abdominal diferencial con la epididimitis.
(hidronefrosis importante), riñón único, disminución de la función 4) El tratamiento óptimo es la cirugía inmediata.
del riñón afectado. 5) La torsión testicular puede producirse en el período neonatal,
pero en estos casos no suele haber ningún riesgo de perder las
Pregunta 71.- R: 2 gónadas.
Este caso clínico sirve para ilustrar el retrato robot del síndrome
hemolítico urémico: Caso clínico de escroto agudo que por las características del cuadro
• Antecedentes de diarrea sanguinolenta (recuerda la asociación (dolor intenso, tumefacción escrotal y ausencia de reflejo cremastéri-
con E. coli O157:H7). co) podemos catalogar de torsión testicular. Se trata de una urgencia
• Clínica de deterioro neurológico (somnolencia, letargia, estupor). quirúrgica (opción 5 incorrecta) porque existe riesgo de pérdida de
• Fracaso renal (se eleva BUN, creatinina, urea). la viabilidad testicular si transcurren más de 6 horas.
• Anemia (recuerda que es microangiopática, y en la extensión
periférica se ven esquistocitos). Pregunta 75.- R: 3
• Trombopenia (clínica de petequias, equimosis). Se da por con- Lactante varón de 10 meses de edad al que en un examen
sumo de plaquetas. rutinario de salud se le descubre una masa en flanco izquierdo,
dura, que sobrepasa línea media. En la ecografía abdominal dicha
Recuerda: masa se corresponde con un tumor sólido localizado en la glán-
• El SHU es la causa más frecuente de insuficiencia renal aguda dula suprarrenal izquierda. ¿Cuál de las siguientes afirmaciones
en niños. es cierta?:

CTO Medicina • C/Núñez de Balboa 115 • 28006 Madrid • Tfno.: (0034) 91 782 43 30/33/34 • E-mail: secretaria@ctomedicina.com • www.grupocto.es
11
Pediatría
Test 2V Distancia
1. La edad inferiro al año empeora el pronóstico.
2. Si presentara metástasis hepáticas estaría contraindicado el
Comentarios
Grupo CTO
CTO Medicina

des por el crecimiento y por la descamación celular. Normalmente


se absorbe un 10% del hierro de los alimentos, con lo cual un niño
tratamiento quirúrgico. debe ingerir de 8 a 10 mg de hierro al día, siendo difícil conseguir
3. Debe realizársele un aspirado de médula ósea como parte del esto con la dieta, sobre todo en los primeros años de vida, en los
estudio de extensión. cuales predominan los alimentos lácteos y otros pobres en hierro. Por
4. El tumor que presenta es más frecuente en pacientes con he- eso, aunque siempre hay que buscar una posible pérdida de sangre
mihipertrofia. ante una anemia ferropénica, desde los 9 a los 24 meses, la causa
5. El empleo de la gammagrafía con metayodobencilguanidina ha más frecuente es la pobre ingesta. Hay que puntualizar sin embargo
sido abandonado por técnicas más específicas. que esta causa es rara en los primeros 4-6 meses, porque aunque la
leche materna y la de vaca son pobres en hierro, los recién nacidos a
Caso clínico compatible con un neuroblastoma (masa abdominal término de peso adecuado nacen con depósitos que suplen la carencia
dura a la palpación, que sobrepasa la línea media, en un niño menor alimentaria. El hierro de la leche materna se absorbe mejor que el
de 2 años). La edad inferior a 1 año es un factor de buen pronóstico de la leche de vaca, y los niños alimentados con lactancia materna
(respuesta 1 falsa). Es un tumor que puede metastatizar en hígado tienen menos necesidades de hierro de otros alimentos que los que
sin que ello contraindique la cirugía (respuesta 2 falsa). Siempre se lactan con leche artificial.
debe hacer aspirado de médula ósea para descartar afectación de la
misma (respuesta 3 correcta). El tumor que se presenta asociado a Las manifestaciones clínicas del déficit de hierro son las mismas
hemihipertrofia no es el neuroblastoma (respuesta 4 falsa) sino el tumor que las producidas por otro tipo de anemias, y además las debidas
de Wilms o nefroblastoma. Se sigue empleando la gammagrafía con a la carencia de hierro en sí: pagofagia (deseo de comer sustancias
MIBG como método diagnóstico y en dosis más altas puede constituir extrañas), disminución de la atención y las capacidades intelectuales
un posible tratamiento (respuesta 5 falsa). (incluso sin anemia), irritabilidad, anorexia.

Pregunta 76.- R: 5 El tratamiento consiste en la administración de suplementos de


Aprovecha esta pregunta para repasar los factores pronósticos hierro en forma de sales ferrosas (6 mg/kg/día de hierro elemental) vía
del neuroblastoma: oral. La respuesta es rápida y ayuda a hacer el diagnóstico en los casos
• La edad menor de un año supone un factor de buen pronóstico. en que no está claro. El primer indicador de mejoría es la clínica en
• La localización abdominal supone un factor de mal pronóstico. cuanto a irritabilidad y apetito del niño, en menos de 24 horas incluso.
• La amplificación del oncogén myc supone mal pronóstico. A las 48 horas aproximadamente se produce una hiperplasia eritroide
• Las metástasis óseas constituyen mal pronóstico. No así las de que puede objetivarse haciendo un examen de sangre periférica. El
hígado, piel o médula ósea. aumento de las cifras de hemoglobina puede aparecer desde 4 días
• El síndrome opsoclono-mioclono no supone un factor de mal después de iniciar el tratamiento, hasta 1 mes después, dependiendo
pronóstico. del déficit de base.

Pregunta 77.- R: 2 Pregunta 80.- R: 1


La madre del enunciado, aunque se expresa toscamente, dice lo Respecto a los tumores de la infancia, señale la afirmación
que tiene que decir. Dice que su niño nació macrosómico (“gordo”) correcta:
y que tenía un onfalocele (“las tripas fuera”), esto es, menciona 1) La leucemia aguda es la patología oncológica más frecuente de
sin hacerlo, pero a gritos, el síndrome de Beckwith-Wiedemann. la infancia.
Este síndrome se asocia con el tumor de Wilms: la masa del flaco 2) El astrocitoma se localiza principalmente en fosa anterior.
derecho del paciente bien puede serlo, ¿no te parece?. Recuerda 3) El tumor sólido extracraneal más frecuente es el nefroblastoma.
algunos datos: 4) No se debe biopsiar un neuroblastoma por el riesgo de sangrado.
• Puede haber hematuria en un 10-25% de los casos. 5) Los niños con tumor de Wilms tienen un aspecto de muy en-
• Para el estudio histológico inicial, se ha de obtener la muestra fermos.
por PAAF, no por biopsia (para no alterar el estadio clínico).
• Las metástasis más frecuentes son las pulmonares. Las leucemias constituyen la patología oncológica más frecuente
• El tumor de Wilms también se asocia a aniridia, hemihipertofia de la infancia (representan el 30% de la oncología infantil), de ellas
y alteraciones genitourinarias. el 97% son agudas (la mayoría linfoblásticas). El astrocitoma es el más
• El estadio V corresponde al tumor bilateral. frecuente de los tumores cerebrales, su localización suele ser infraten-
torial (en fosa posterior). El tumor sólido extracraneal más frecuente es
Pregunta 78.- R: 2 el neuroblastoma, se localiza principalmente en el abdomen.
Señale la afirmación FALSA respecto a la PTI:
1) Existe un importante descenso del número de plaquetas circu- El nefroblastoma no se debe biopsiar (sí el neuroblastoma) porque
lante, aunque el número de megacariocitos en médula ósea es la rotura de la cápsula renal cambia el estadio. Los niños afectos de
normal. tumor de Wilms en general son algo mayores que los afectos por
2) Cursa con importante esplenomegalia. neuroblastoma y parecen menos enfermos.
3) La complicación más grave de la enfermedad es la hemorragia
intracraneal. Pregunta 81.- R: 4
4) En el tratamiento se emplea gammaglobulina i.v. y los corticoides. Con respecto al diagnóstico del niño HIV positivo, señale cuál
5) El tiempo de hemorragia y el tiempo de retracción del coágulo de las siguientes afirmaciones es FALSA:
están alterados. 1) En lactantes mayores de 18 meses, el diagnóstico se hace igual
que en adultos.
La forma aguda de la PTI suele ser una enfermedad infantil que 2) La edad de presentación, el recuento de linfocitos CD4/CD8 y
aparece tras procesos víricos de vía respiratorias altas. Existe un la carga viral son factores que condicionan el pronóstico.
importante descenso del número de plaquetas con alteración del 3) En un niño mayor de 18 meses puede hacerse el diagnóstico de
tiempo de hemorragia y el tiempo de retracción del coágulo. La infección por VIH, si existen dos determinaciones positivas al
mayor parte de los casos tienen una recuperación espontánea con realizar el cultivo viral, la PCR o la determinación del Ag p24.
escasa recurrencia y mortalidad. Sin embargo, pueden aparecer com- 4) Al nacimiento, todos los hijos de madre VIH+ tienen anticuerpos
plicaciones hemorrágicas graves, como la hemorragia intracraneal, (Acs) tipo Ig A frente al HIV.
que requieran tratamiento con gammaglobulina i.v. y corticoides. Es 5) Para diagnosticar la serorreversión de los Acs en un niño entre
clave recordar que la esplenomegalia es un dato en contra de la PTI los 6 y 18 meses, son necesarios dos test de ELISA negativos.
(respuesta 2 falsa).
El diagnóstico del VIH en el RN de madre VIH+ se ve dificultado
Pregunta 79.- R: 3 porque al nacimiento estos niños presentan Acs tipo IgG frente al
La anemia ferropénica es la enfermedad hematológica más fre- VIH (respuesta 4 incorrecta) procedentes de la madre que pasan
cuente en la infancia, y es la primera sospecha diagnóstica en un niño a través de la placenta. Casi todos estos niños son seropositivos al
con anemia microcítica sin otros indicadores de patologías específicas. nacer y sólo un 15-30% están verdaderamente infectados. El resto
Los niños, deben absorber de la dieta 1 mg de hierro diario para de los niños seropositivos negativizan estos anticuerpos entre los 9 y
mantener un balance positivo, y poder hacer frente a sus necesida- 18 meses de vida.

CTO Medicina • C/Núñez de Balboa 115 • 28006 Madrid • Tfno.: (0034) 91 782 43 30/33/34 • E-mail: secretaria@ctomedicina.com • www.grupocto.es
12
Comentarios
Grupo CTO
CTO Medicina

El método de detección de IgG utilizado en adultos no es un


método fiable de diagnóstico en menores de 18 meses; por eso, hay
Test 2V Distancia

Criterios mayores
Pediatría

Criterios menores
que recurrir a procedimientos diagnósticos más complejos como el
cultivo viral, la PCR y el antígeno p24 en suero o plasma. Carditis Fiebre
Poliartritis migratoria Artralgia
Fiebre reumática previa
Pregunta 82.- R: 4 Eritema marginado
Elevación de reactantes de fase
Pregunta muy fácil, sobre todo porque aunque de entrada por el Corea aguda (VSG, PCR)
enunciado no tuvieseis el diagnóstico, es fácil rechazar el resto de las Nódulos subcutáneos Intervalo P-R prolongado en el ECG
respuestas, con las que no coincide el cuadro clínico descrito.
Aprovechamos para repasar las características de la púrpura de Pregunta 84. Criterios de Jones de la fiebre reumática
Shönlein-Henoch, una patología propia de la infancia. Se desconoce
su causa, pero es frecuente que vaya precedida 1-2 semanas antes Celulitis Celulitis Trombosis del
de una infección de vías respiratorias altas. Puede aparecer de forma Síntoma
preseptal orbitaria seno cavernoso
súbita o insidiosa, siendo las manifestaciones cutáneas las que dan
el diagnóstico: exantema inicialmente maculoso, después palpable, Edema palpetral
SÍ SÍ SÍ
rosado, que al principio desaparece a la presión para que pronto las inflamatorio
lesiones sean una púrpura palpable violácea que se vuelve marronácea Exoftalmos NO Sí (monolateral) SÍ (bilateral)
y desaparece finalmente.
Dolor local a la
SÍ Importante Tolerable
La distribución es típica: en las partes más declives, por tanto en presión
las extremidades inferiores. Puede haber angioedema local asociado Dolor con la
a las lesiones purpúricas. Por afectación vascular de pequeño vaso, NO SÍ SÍ
movilidad ocular
puede haber artritis, afectación intestinal, y renal. La aparición del
exantema a veces se produce en brotes, con intervalos de entre Repercusión
Leve Moderada Muy grave
unos días y 3-4 meses, aunque es excepcional que se prolonguen general
más de 1 año.

Pregunta 83.- R: 3 Pregunta 85.- R: 5


El cuadro clínico que presenta nuestro paciente es una tos ferina. El • La máxima contagiosidad en el sarampión se da 5 días antes de
caso narra un lactante de 10 meses que presentó un cuadro catarral la aparición de exantema.
inespecífico (fase catarral) que duró dos semanas, y que evolucionó • Los casos de sarampión en menores de 6 meses son raros, por la
hasta una fase de estado (fase paroxística). En ésta, destacan los accesos protección que ejercen los anticuerpos maternos.
de tos que acaban en gallo (ruido inspiratorio) o en vómitos. • Las manchas de Koplik (máculas blanquecinas en la mucosa sub-
En los lactantes más pequeños, los accesos de tos pueden acabar yugal opuesta a los molares inferiores) son el enantema patogno-
en pausas de apnea, con lo cual pueden resultar letales (opción 1, mónico del sarampión.
correcta). Otras posibles complicaciones son: OMA, neumonías, • En la fase prodrómica del sarampión se suele detectar, acompañando
alteraciones de la función pulmonar de carácter transitorio (opción al cuadro catarral y a la conjuntivitis fotofóbica, una fiebre moderada.
5, correcta), convulsiones y hemorragias conjuntivales por esfuerzo Ésta se eleva bruscamente cuando aparece el exantema.
tusígeno (opción 2, correcta). • El exantema sigue un curso descendente y centrífugo. Se inicia
La leucocitosis a expensas de linfocitosis en el plano analítico en cara y cuello, y progresa caudalmente hasta afectar palmas y
es muy característica. Se ha de tratar con eritromicina a todos los plantas. Se resuelve a través de una descamación furfurácea en el
pacientes con tos ferina y a sus familiares, para eliminar la cadena mismo orden de aparición.
epidemiológica de portadores (opción 3, falsa: no se emplean Igs).
Los elementos transmisores suelen ser adultos paucisintomáticos y Pregunta 86.- R: 2
tosedores crónicos (opción 4, correcta). Principales complicaciones del sarampión:
• Otitis media aguda.
Pregunta 84.- R: 4 • Neumonía. Lo más frecuente es la neumonía bacteriana secun-
El cuadro clínico que se presenta es una enfermedad exantemá- daria. La neumonía por el virus del sarampión, o neumonía de
tica con faringoamigdalitis. Se te tienen que venir, con sólo estos células gigantes de Hecht, es menos frecuente, y se da sobre
datos, dos enfermedades a la mente: escarlatina y mononucleosis todo en inmunodeprimidos.
infecciosa. La distinción entre ambas es muy sencilla, pues el caso te • Alteraciones neurológicas. Con gran frecuencia aparecen altera-
da muy buenas pistas: ciones inespecíficas en el EEG sin clínica asociada. La aparición
• Petequias en paladar: muy típicas de las faringoamigdalitis de encefalomielitis (1/1000 casos) es independiente de la seve-
estreptocócicas (escarlatina, por tanto). ridad y extensión del exantema. La panencefalitis esclerosante
• Lengua en frambuesa: a favor de la escarlatina. También se ve subaguda de Dawson, cuya expresividad EEG se da a través
en la enfermedad de Kawasaki. de los complejos de Rademeker, consiste en el deterioro del
• Exantema confluente en pliegues: líneas de Pastia, típicas de la rendimiento escolar, la personalidad y la función intelectiva, y
escarlatina. afecta a niños en edad escolar que padecieron el sarampión a
• Eritema facial que respeta el triángulo nasogeniano: facies de edades tempranas.
Filatov, típica de la escarlatina. Recuerda otro exantema facial: • El sarampión es anergizante, y puede provocar la reactivación
el del eritema infeccioso, con afectación “en bofetón de ambas de una tuberculosis latente.
mejillas”.
Pregunta 87.- R: 2
Nos decantamos, pues, por la escarlatina: En relación con las enfermedades exantemáticas, recuerda:
• La fiebre y la contagiosidad remiten en los primeros 1-2 días • El pródromos de cualquier exantemática es inespecífico, y
tras la instauración de la antibioterapia. consiste en un cuadro catarral con fiebre.
• El exantema de la escarlatina respeta palmas y plantas (recuer- • El exantema ya es otro cantar. En este caso es rosado, no con-
da que afectan a palmas y plantas: sarampión, fiebre botonosa fluente (descartamos por ello el sarampión, que es eritematoso y
y lúes). confluente), no pruriginoso (con lo cual descartamos la varicela).
• La escarlatina secundaria a faringoamigdalitis puede complicarse • Se acompaña de conjuntivitis (con ello descartamos la escarlatina
con glomerulonefritis y con fiebre reumática (que nunca será y el exantema súbito). Los cuadros exantemáticos con conjun-
secundaria a estreptococias cutáneas). tivitis pueden ser debidos a: sarampión, rubéola, adenovirus y
• La presencia de S. pyogenes en un cultivo de faringe confirma enfermedad de Kawasaki.
el diagnóstico. • Asocia adenopatías dolorosas posteriores. Esto es muy típico
• El tratamiento de elección consiste en administrar penicilina de la rubéola. Pueden aparecer adenopatías posteriores en
oral durante 10 días. En alérgicos se usará eritromicina, y en la mononucleosis infecciosa (si bien lo más frecuente es que
pacientes de los cuales se dude el cumplimiento terapéutico, sean laterocervicales bilaterales y múltiples. Recuerda que en
se recurrirá a la penicilina benzatina i.m. en dosis única. la mononucleosis puede aparecer exantema (por antibióticos),

CTO Medicina • C/Núñez de Balboa 115 • 28006 Madrid • Tfno.: (0034) 91 782 43 30/33/34 • E-mail: secretaria@ctomedicina.com • www.grupocto.es
13
Pediatría
Test 2V Distancia
esplenomegalia y faringoamigdalitis (si bien esta es exudativa,
no como la de nuestro caso clínico).
Comentarios
Grupo CTO
CTO Medicina

Pregunta 92.- R: 3
Un niño de 13 años presenta dolor y tumoración en la mandíbula
• Aparace un enantema: manchas rojas en el velo de paladar. Estas derecha, que borra ángulo y ha ido aumentando rápidamente en
son las características manchas de Forcheimer de la rubéola. pocas horas. Además, presenta fiebre de 38,5 C y cefalea. Señale
º

Recuerda que las faringoamigdalitis estreptocócicas pueden la afirmación cierta:


asociar petequias en el paladar. 1) El 95% de las infecciones por este paramixovirus producen clínica.
2) En más del 65% de los enfermos se detecta pleocitosis en el
Por tanto, la respuesta correcta es la opción 2, rubéola: exantema LCR, aunque no presenten manifestaciones clínicas de menin-
rosado con adenopatías posteriores, manchas de Forcheimer y con- goencefalitis.
juntivitis no puede ser otra cosa. 3) Con frecuencia se acompaña de pancreatitis clínica.
4) La tumefacción parotídea es indolora y existen algunos signos
Pregunta 88.- R: 5 de inflamación de la piel suprayacente.
Niño de catorce meses traído a Urgencias por su madre al 5) En un elevado porcentaje de los varones aparece esterilidad
observar desconexión del medio, rigidez tónica, versión ocular y tras la orquitis postparotiditis.
cianosis perioral, de tres minutos de duración, con somnolencia
posterior durante cinco minutos. Al llegar a Urgencias se observa Los datos clínicos de dolor, tumefacción que borra el ángulo de la
temperatura de 39ºC. La exploración neurológica es normal y la mandíbula y fiebre en un niño, deben hacernos pensar en una paro-
otoscopia muestra signos de otitis media serosa. La madre refiere tiditis. Hasta el 40% de casos de parotiditis cursan de forma subclínica
que a la edad de seis meses tuvo un episodio similar, también (respuesta 1 falsa). La piel suprayacente a la glándula inflamada no está
coincidiendo con fiebre. ¿Cuál sería la actitud más correcta?: afectada, y la palpación suele ser dolorosa (respuesta 4 falsa). En más
1) Observación durante 24 horas e iniciar tratamiento con carba- del 65% de los pacientes se detecta pleocitosis en el LCR aunque no
macepina. presenten clínica de meningoencefalitis (respuesta 2 cierta). Compli-
2) Realizar TC craneal y punción lumbar. caciones como la pancreatitis clínica y la esterilidad total residual en
3) TC craneal, punción lumbar y hemocultivos seriados. varones postpúberes son raras (respuestas 3 y 5 falsas).
4) EEG e iniciar tratamiento con ácido valproico.
5) Observación, tratamiento antitérmico y seguimiento ambulatorio. Pregunta 93.- R: 2
Este caso clínico presenta:
El cuadro descrito se corresponde con una convulsión febril, causa • A un adolescente.
más frecuente de convulsión en el niño. Aparecen en algunos niños • Con un proceso febril con adenopatías,
entre los 6 meses y los 5 años de edad coincidiendo con episodios • Astenia,
febriles en los que la temperatura se eleva por encima de los 38ºC. Las • Faringoamigdalitis exudativa,
convulsiones febriles típicas duran menos de 10 minutos y suelen ser • Y esplenomegalia.
tónico-clónicas, sin suponer un aumento del riesgo de padecer epilepsia
en la edad adulta. La actitud más correcta para su manejo es el control Este es el “retrato robot” de la mononucleosis infecciosa, cuyo
de la temperatura corporal con antipiréticos como el paracetamol. agente etiológico es el virus de Epstein-Barr. Recuerda que hay síndro-
mes mononucleósicos producidos por otros gérmenes (CMV, que cursa
Pregunta 89.- R: 1 con faringoamigdalitis no exudativa y adenopatías de pequeño tamaño;
• La artritis por rubéola suele afectar a las pequeñas articulaciones (sobre Toxoplasma gondii, que cursa sin faringitis; rubéola; HIV). El VEB tiene
todo, metacarpofalángicas) de postpúberes de sexo femenino. un claro tropismo por los linfocitos B. Recuerda que los clásicos linfocitos
• La encefalitis rubeólica es 6 veces menos frecuente que la saram- atípicos de la mononucleosis son linfocitos T activados para destruir a los
pionosa. linfocitos B colonizados por el virus. El VEB provoca en gran número de
• La profilaxis con gammaglobulina se ofrecerá a gestantes no vacu- ocasiones una hepatitis subclínica, con lo cual se acompaña de elevación
nadas que hayan contactado con el virus salvaje, como alternativa de las transaminasas. Si a un paciente con mononucleosis se le trata con
al aborto terapéutico (alto riesgo de rubéola congénita). beta-lactámicos, aparecerá un exantema inespecífico en un 80% de las
• Los pacientes con rubéola congénita presentan viruria mantenida ocasiones. La determinación de anticuerpos heterófilos puede ayudar al
durante los primeros meses de vida. diagnóstico de esta enfermedad, si bien el dato analítico más específico
• La probabilidad de padecer una rubéola asintomática es el doble es la determinación serológica de la IgM anti-VEB.
que la de padecer la enfermedad clásica.
Pregunta 94.- R: 5
Pregunta 90.- R: 3 ¿Cuál de las siguientes es una característica propia del SIDA
Esta pregunta es esencial. Ha caído muchas veces en el MIR. Re- infantil respecto al SIDA en los adultos?
cuerda que un lactante que padece un cuadro febril de 3-4 días de 1. Mayor tiempo de incubación.
duración sin foco aparente, y que al solucionarse da paso a la aparición 2. Hipergammaglobulinemia tardía.
de un exantema maculoso de predominio en tronco y miembros supe- 3. Menor número de infecciones bacterianas.
riores, sólo puede ser un exantema súbito, generalmente producido 4. Mayor linfopenia.
por virus herpes tipo 6. No dejes que te despiste la analítica: en la 5. Menor número de infecciones oportunistas.
fase aguda el hemograma muestra leucocitosis con neutrofilia (como
si fuese una infección bacteriana). En el período exantemático ya se Recuerda las diferencias más significativas del SIDA infantil respecto al
ve el patrón típico de leucopenia con predominio de linfocitos, tan del adulto: El período de incubación es mas corto en los niños y la hiper-
típico de las viriasis. Tampoco dejes de recordar que la complicación gammaglobulinemia más precoz. El número de infecciones bacterianas y
más frecuente (crisis febriles) se da en la etapa pre-exantema. de alteraciones neurológicas es mayor que en los adultos. Sin embargo,
la inversión del cociente CD4/CD8 es más tardía, la linfopenia es menor
Pregunta 91.- R: 4 y las infecciones oportunistas son menos frecuentes que en los adultos
Un exantema que no tiene pérdida es el variceloso: su carácter (respuesta correcta 5), pero de mayor agresividad.
polimorfo (con máculas, pápulas, vesículas, pústulas y costras coexis-
tiendo sobre la piel del paciente), la afectación de mucosas (oral, anal, Pregunta 95.- R: 3
genital) y su cualidad pruriginosa lo hacen singular y perfectamente Respecto al Síndrome de Muerte Súbita Infantil o del Lactante (SMSL),
reconocible. La opción correcta, por tanto, es la 4. señale cuál de los siguientes NO constituye un factor de riesgo:
1) Antecedentes familiares.
Opción 1. La primoinfección herpética (por herpes I) sólo afecta 2) Prematuridad.
a la mucosa oral, en forma de gingivoestomatitis. Opción 2. La enfer- 3) Posición para dormir en decúbito supino.
medad mano-pie-boca se da por enterovirus, y cursa con la aparición 4) El sexo masculino.
de vesículas (sólo vesículas) en los territorios referidos. Opción 3. La 5) El tabaquismo materno.
erupción variceliforme de Kaposi produce una afectación generali-
zada secundariamente a una infección por virus herpes, y afecta a El SMSL, muerte repentina e inexplicable de un niño menor de un
inmunodeprimidos o pacientes con dermatitis atópica (la paciente año, supone la causa principal de muerte entre el mes de vida y el año
del enunciado es una niña sana). Opción 5. La herpangina cursa con de edad en los países desarrollados. Este síndrome se correlaciona con la
vesículas en los pilares anteriores de la faringe. posición de decúbito prono y lateral para dormir (opción 3 incorrecta).

CTO Medicina • C/Núñez de Balboa 115 • 28006 Madrid • Tfno.: (0034) 91 782 43 30/33/34 • E-mail: secretaria@ctomedicina.com • www.grupocto.es
14
Comentarios
Grupo CTO
CTO Medicina

Tras este factor posicional, el tabaquismo materno es actualmente el


principal factor de riesgo del SMSL. Otros factores son la prematuridad,
Test 2VDistancia
Pregunta 100.- R: 5
Pediatría

En relación con el trastorno por hiperactividad con déficit de


el antecedente de hermano muerto por SMSL, sexo masculino, etc. De- atención recuerda:
bido a la ausencia de tratamiento es muy importante la prevención con • Parece haber un componente heredofamiliar importante.
medidas como la posición de decúbito supino para dormir y la lactancia • Se ha demostrado la asociación entre el gen del receptor dopami-
materna (que se considera el factor preventivo más importante). nérgico D4 y un fenotipo de THDA.
• La proporción varones/niñas es 4/1.
Pregunta 96.- R: 3 • Hay pruebas de imagen que han documentado patología estruc-
El Fallot es una cardiopatía cianosante que cursa con disnea tural en los niños con THDA en el circuito prefrontal-estriado-
que clásicamente mejora con la posición de cuclillas (squatting). talamocortical.
El hecho determinante de la clínica de la tetralogía de Fallot es • Para su tratamiento no se emplean sedantes, sino psicoestimulantes
el grado de obstrucción a la salida del ventrículo derecho. Al (para mejorar la atención) y psicoterapia.
haber obstrucción a la salida del ventrículo derecho, en el ECG
veremos signos de hipertrofia compensatoria del mismo. La alte-
ración radiológica del Fallot es el “corazón en zueco”, con el ápex
levantado, la silueta ventricular derecha aumentada de tamaño
y los pulmones hipoémicos. Recuerda que se ausculta un soplo
sistólico en foco pulmonar.

Pregunta 97.- R: 2
Las cardiopatías congénitas son cuadros complejos de estudiar,
pero al menos queremos que memorices pequeños detalles, como
el de la radiología típica de la coartación aórtica. Se aprecian mues-
cas costales, debidas al engrosamiento de las arterias costales por el
aumento de la presión arterial.

Pregunta 98.- R: 4
Hasta un 5% de los lactantes cuya edad está comprendida entre
los 6 meses y los 4 años pueden desarrollar crisis febriles.

Cuando a un Servicio de urgencias pediátricas llega un niño pe-


queño convulsionando, hay que tomarle la temperatura e interrogar
a los padres sobre síntomas que anuncien una infección (tos, rinorrea,
vómitos, diarrea), pues la causa más frecuente de convulsión a estas
edades son las crisis febriles.

Las crisis febriles típicas duran menos de diez minutos y son


generalizadas. Ante síntomas de focalidad, habrá que hacer una
exploración neurológica más detallada, que incluirá en todos los
casos EEG.

La complicación más frecuente de las crisis febriles es la recurrencia,


en un 40-50% de los casos.

Simples o típicas Complejas o atípicas

Frecuencia 85 - 90 % 10 - 15 %

Duración < 15 minutos > 15 minutos

Número por cada > 1 cada 24h o en el


1
proceso febril mismo proceso febril
Prolongado, parálisis
Período postcrítico Corto
de Tood
Riesgo de epilepsia
Poco Más riesgo
ulterior
Pregunta 98. Clasificación de las convulsiones febriles

Pregunta 99.- R: 3
En esta pregunta vamos a comentar el “retrato robot” de un cuadro
epiléptico propio de la infancia: el síndrome de West. La conjunción
clave es:
• Epidemiología: Lactante.
• Clínica: Espasmos (los más frecuentes son los mixtos, con flexión
y extensión).
• Trazado electroencefalográfico intercrítico: hipsarritmia (para que lo
recuerdes, es un patrón caótico, “similar a la fibrilación ventricular
del ECG”).

El tratamiento de primera elección del síndrome de West es el


ACTH; de segunda elección se recurre al ácido valproico. En casos
rebeldes, se pueden emplear combinaciones terapéuticas o los nuevos
anticomiciales.

CTO Medicina • C/Núñez de Balboa 115 • 28006 Madrid • Tfno.: (0034) 91 782 43 30/33/34 • E-mail: secretaria@ctomedicina.com • www.grupocto.es
15

Vous aimerez peut-être aussi